Sample Study Material: IIT-JAM Physics

You might also like

Download as pdf or txt
Download as pdf or txt
You are on page 1of 78

Sample Study Material

IIT-JAM Physics

Holistic Learning Platform for Higher Education Exams

www.eduncle.com info@eduncle.com 1800-120-1021


Unit-3 Mechanics

1. Introduction to Mechanics

Mechanics, is a branch of science, related with the motion of bodies under the influence of forces, including the special
case in which a body remains at rest.
Scientists Galileo, Kepler and Newton developed Classical Mechanics, during the early morning period. It is branch
of classical physics that deals with particles that are either at rest or are moving with velocities less than the speed
of light (c = 3 × 108 m/sec.)

Branches of Mechanics
(i) Static : This branch deals with the study of object at rest or in equilibrium, even when they are under the action
of several forces.
(ii) Kinematics : This branch relate to the study of motion of objects without considering the cause of motion.
(iii) Dynamics : It is branch of mechanics, that deals with the study of motion of objects, taking into consideration
the cause of their motion.
 Classical mechanics can be divided in different types, according to different-different explanations given by
scientists, like Newtonian mechanics, Lagrangian mechanics, Hamilton’s mechanics etc.
 For the description of the small scale phenomena of atomic and nuclear physics, a new theory, known as
Quantum mechanics has been formulated.

General Approach for Solving Problems in Physics


General Approach

Start with Equation of Motion Start with Conservation Laws

In general we solve these problems mostly by using Lagrangian and Newtonian mechanics.
In this chapter we will deal with newtonian mechanics and solve the problems using equation of motion.
Mass is Absolute
Mechanics
Newtonian

Length is Absolute

Time is Absolute

2. Force

Force can be defined as an external agent capable of changing the state (in rest or in motion) of a particular body.

Contact Us : Website : www.eduncle.com | Email : support@eduncle.com | Call Us : 7231923276 3.1


Mechanics

2.1 Types of Force


There are two types of Forces based on their applications :
Types of Force

Contact Force Non-Contact Force

1. Contact Force : Force which act on a body either directly or through a medium is called contact force. E.g..
Frictional force
2. Non-Contact Force : Force which act through space without making any direct contact with the body is called
non-contact force. E.g.. Gravitational Force
Formula, unit and dimension:
That physical parameter which produces or tends to produce an acceleration in a particle, is defined as force.
 
F  ma
1 Unit of force = Newton in MKS system = Dyne in CGS system
1 Newton = 105 Dyne
Dimension of force is M1L1T-2.

Can nonliving bodies exert a force ?


Yes, they can. If we stand in a great sto rm, we feel that the wind is exerting a force
on us. When we suspend a heavy block from a rope, the rope holds the block just
as a man can hold it in air. When we comb our dry hair and bring the comb close to
small pieces of paper, the pieces jump to the comb. The comb has attracted the
paper pieces i.e. the comb has exerted force on the pieces. When a cork is dipped
in the water, we have to push it downward. We say that water exerts a force on the
cork in the upward direction.

Normal Force
The word normal means perpendicular to surface. The normal force can be less than the object’s weight if the subject
is on an incline. It is represent by N.
Here weight is force due to gravity.

Example
The person holding the bag of dog food must supply an upward force Fhand equal in magnitude and opposite in direction
to the weight of the food w.
When the dog food is placed on it, the card table Sags. Elastic restoring forces in the table grow as it sags until they
supply a force equal in magnitude and opposite in direction to the weight of the load.

Contact Us : Website : www.eduncle.com | Email : support@eduncle.com | Call Us : 7231923276 3.2


Mechanics

Fhand

N
Bow Wow Chow
Bow Wow Chow

W
W

(a) (b)

Fhand N

W W

Fig. : Free-Body Diagrams

Tension
In a tug of war, two persons hold the two ends of a rope and try to pull the rope on their respective sides. The rope
becomes tight and its length is slightly increased. In many situations this increase is very small and goes undetected.
Such a stretched rope is said to be in a state of tension.
A Tension is a force along the length of a medium, especially a force carried by a flexible medium, such as a rope
or cable.
This is an example of Newton’s third law. The rope is the medium that carries the equal and opposite force between
the two objects. The tension anywhere in the rope between the hand and the mass is equal. Once you determine the
tension in one location, you can determine the tension all locations along the rope.

T
m
W

Fig. : Tension in The Rope

Tension in the rope must be equal to the weight of the supported mass, we can prove this using Newton’s second
law. If the 5.00-kg mass in the figure is stationary, then its acceleration is zero, and thus Fnet = 0. The only external
forces acting on the mass are its weight w and the tension T supplied by the rope.

Contact Us : Website : www.eduncle.com | Email : support@eduncle.com | Call Us : 7231923276 3.3


Mechanics

Fundamental Forces
There are four fundamental forces of nature :
1. Gravitational Force : Weakest Force; Infinite Range
2. Weak Nuclear Force : Next weakest, but short range
3. Electromagnetic Force : Stronger, with infinite range
4. Strong Nuclear Force : Strongest but short Range

Electro- Weak Strong


Gravitation
magnetism Interaction Interaction

P
P P N
N P N
P P
P N
N P
N

Fig. : The Four Fundamental Forces of Nature

3. Newtons Laws of Motion

“ I can calculate the motions of the heavenly bodies, but not the madness of people”
- Newton

Newton’s laws of motion are stated in the following forms :


(i) "A body at rest or uniform motion will continue to be at rest or uniform motion until and unless a net external
force (F) acts on it".
(ii) “The time rate of change of total momentum of particle is proportional to the net external force acting on it
and is in the direction of the force.”
(iii) “To every action there is always an equal and opposite reaction” or “the mutual actions of any two bodies
are always equal and oppositely directed along the same straight line”.
Assumptions : (point particle) (only position, no linear dimension)
Reference Frame : Valid in inertial frame only, when no fictitious force is present. (which we have described in topic
fictitious and pseudo forces)
Space : Euclidean space
Time : Constant time in all reference frame
Relativity : Galilean relativity (we will discuss it in Modern Physics Unit)

Force in Terms of Momentum


Linear Momentum
 
p  mv
is given by Newton’s second law which can be expressed as
 dp
F
dt

 dp d  
or F  k  k (mv)  kma
dt dt
Contact Us : Website : www.eduncle.com | Email : support@eduncle.com | Call Us : 7231923276 3.4
Mechanics

where, m = mass of particle



v = velocity
t = time

p = momentum

a = acceleration
where k is the constant of proportionality. This constant can be chosen to be equal to unity by defining the unity of
the force as that force which while acting on a body of unit mass produces a unit acceleration.
 
F  ma
This formula cannot be applicable for variable mass.

1 st Law
Special
1st Law : Case 2n d Law :
Inertia F = ma
All
Matter

Ft = M v F1 = – F2

3 rd Law :
Equal and Opposite

Fig. : Newton’s Laws of Motion


in Venn Digram
Law Statement Possible Formula Salient features Examples
(A) First Law A body continues to be If F = 0, (i) This law defines the (i) When a stationary
(Law of in its state of rest or V = constant a = 0 qualitative definition of vehicle suddenly moves
Inertia) uniform motion until force. then the passenger inside
and unless it is acted (ii) The body opposes any the vehicle falls
upon by some external external change in its state of backwards.
force. rest or uniform motion. (ii) When a carpet or a
(iii) It is also known as the blanket is beaten with a
law of inertia of Galilio. stick then the dust is
(iv) This law can be divided separated out from it.
in two parts (A) Law of (iii) A coin on the
inertia of stationary state (B) cardboard placed on glass
Law of inertia of the state of falls into the glass if it is
motion given a sudden jerk.
(B) Second this law gives  dp (i) The change in momentum (i) Different acceleration
(i) F 
Law dt takes place in the direction of are produced in different
 mv  mu applied force i.e., a and F are bodies of same mass but
(ii) F 
t in the same direction.(ii) different size.
This law gives magnitude of
force.
(C) Third Law To even action force   (i) Action and reaction are (i) Walking of a person on
F12  F21
or Law of there is an equal and mutually opposite and act on the road.
Action and reaction force. two different bodies. (ii) Jerk produced in a gun
Reaction (ii) When a force acts on a when a bullet is fired from
body then there action acts it.
normally to the surface of (iii) Jumping of a man
the body. from a boat on the bank of
a river.
Contact Us : Website : www.eduncle.com | Email : support@eduncle.com | Call Us : 7231923276 3.5
Mechanics

Failures of Newton’s Laws of Motion


1. Newton’s laws doesn’t apply for Quantum Mechanical cases.
2. Newton’s laws do not directly apply in non-inertial frame, however when pseudo forces are included then
newton’s laws apply in non-inertial frames. (we have explained it in topic Inertial and non-inertial frame of
reference)
3. Newton’s third law strictly apply only for mechanical cases (mass related cases). Newton’s third law of
motion cases to hold good for particles of atomic dimensions, for which simultaneous measurement of two
forces is almost impossible (refer heisenberg uncertainity principle)
 
4. The relation F  ma does not hold good for cases where m is not constant.

3.1 Newton’s Equations of Motion


Newton’s Equations of Motion
  1
Ist equation, v  u  at IInd equation S  ut  at 2
  
2

IIIrd equaion v 2  u 2  2aS
 

where u = Initial velocity


t = time
v = final velocity
S = distance travelled by particle
a = acceleration

How to use Newton’s equation of motion ?


If a problem is related to form, then use following steps :
1. Draw a simple and neat diagram of the system. Isolate the object whose motion is being analyzed. Draw
a free body diagram (fbd) for this object. Assign a coordinate system (Cartesian Preferably)
2. Resolve all forces in x and y direction
3. Then use Newton’s equation of motion
Fx = max,
Fy = may
Fx = Net force in x-direction
Fy = Net force in y-direction
dv x d2x
Use a x  or ax 
dt dt 2
dv x
or, a x  v x
dx
dv x
ax 
dt
dv x dx

dx dt
dv x
 vx
dx

Contact Us : Website : www.eduncle.com | Email : support@eduncle.com | Call Us : 7231923276 3.6


Mechanics

4. V(x) = f(t)

dx
 f (t)
dt

x = f1(t)
5. After this solve above differential equations (use techniques of differential equations)
6. If acceleration ax = constant
then we can avoid above two steps, by using newton’s equation of motion.

3.2 Result of Force Applied in Various States


let us assume a particle P on which we are applying force from different angles, as a result of that particle changes
its path. We consider following cases-
Case-I if force is applied in direction of velocity.

F P V

In case (fig.) only the magnitude of velocity of the particle changes whereas its direction remains same.
Consequently the path of particle is a straight line.
Case-II if force is applied in direction perpendicular to velocity

P v
r
F

Fig.
In case (fig.) only the direction of velocity of the particle changes whereas its magnitude remains
constant. Consequently the path of the particle is a circle.
Case-III If force is applied at an angle from direction of velocity

P v
F

Fig.
In case (fig.) both the magnitude as well as direction of velocity of the particle change. Consequently
the path of motion of the particle is a helical.

Example : The speed of sound propagation in air as a function of temperature T is given by v = aT, where a is a
constant of appropriate dimensions. Calculate the time taken for a sound wave to travel a distance L between two point
A and B, if the air temperature between the points varies linearly from T1 to T2. (IIT JAM Physics 2003)
Solution : Velocity at temperature T1
V1 = T1
Velocity at temperature T2
V2 = T2
form equation of motion,
v2  u 2
a
2s

Contact Us : Website : www.eduncle.com | Email : support@eduncle.com | Call Us : 7231923276 3.7


Mechanics

 2 (T22  T12 )
a
2L
and from
vu
t
a
Here t is time taken by sound wave to travel a distance ‘L’ from A to B.
(T2  T1 )
Time t 
 2 (T22  T12 )
2L
2L
t
(T2  T1 )

Drag Force (Resistive Force)


A particle of mass m projected horizontally in a resistive medium with initial velocity v the drag or resistive force is
equal to the
F   kv

Example : A 77-kg person is parachuting and experiencing a downward acceleration of 2.5 m/s2 shortly after opening
the parachute. The mass of the parachute is 5.2 kg (a) Find the upward force exerted on the parachute by the air.
(b) Calculate the downward force exerted by the person on the parachute.

Air Resistance
PUSH
or “Drag”

PULL Gravity

Force acting on the body (F) is equal to the product of mass of the body (m)
and acceleration of the body (a).
So, F = ma
From equation F = ma, the acceleration (a) of the body would be,
a = F/m
c o n c e pt Weight W of the object is equal to the mass m of the object times of the free
fall acceleration g.
W = mg
Solution :
(a) The net force Fnet on the system is equal to the sum of force exerted by the person and force exerted by the
parachute.
So, Fnet = (mpe + mpa) (a)
Here, mpe is the mass of person, mpa is the mass of parachute and a is the downward acceleration.
To obtain the net force Fnet on the system, substitute 77 kg for mpe, 5.2 kg for mpa and –2.5 m/s2 for a in the equation
Fnet = (mpe + mpa) (a),
= (–205.5 kg.m/s2) (1 N/1 kg.m/s2) = –205.5 N
The weight W of the system will be,
W = (mpe + mpa) (g)

Contact Us : Website : www.eduncle.com | Email : support@eduncle.com | Call Us : 7231923276 3.8


Mechanics

To obtain the weight W of the system, substitute 77 kg for mpe, 5.2 kg for mpa and 9.81 m/s2 for free all acceleration
g in the equation W = (mpe + mpa) (g),
W = (mpe + mpa) (g) = (77 kg + 5.2 kg) (9.81 m/s2)
= (810 kg.m/s2) (1N = 1 kg.m/s2) = 810 N
If P is the upward force of the air on the system (parachute) then,
P = Fnet + W = (–210 N) + (810 N) = 600 N
From the above observation we conclude that, the upward force exerted on the parachute by the air would be 600 N.
(b) The net force Fnet on the parachute will be,
Fnet = mpa a
Here, mpa is the mass of parachute and a is the downward acceleration.
To obtain the net force Fnet on the parachute, substitute 5.2 kg for mpa and –2.5 m/s2 for a in the equation Fnet = mpa a,
Fnet = mpa a = (5.2 kg) (–2.5 m/s2)
= (–13 kg.m/s2) = –13 N
The weight W of the parachute will be,
W = (mpa) (g)
= (5.2 kg) (9.81 m/s2) = (51 kg.m/s2) (1 N=1 kg.m/s2)
= 51 N
If D is the downward force of the person on the parachute then,
D = –Fnet – W + P
= –(–13 N) – (51 N) + (600 N) = 562 N

Example : A particle (m) is dropped near earth. In addition to gravity (g = const) a drag force ±kv acts on the particle.
Find speed after time t. Plot speed v/s time.
Solution : Drag force always acts opposite to v (velocity)
x=0
t=0 v=0

kv
v

mg

+X
Free Body Diagram (FBD)
DRAG force always acts in opposite to speed :
 In free body diagram, we do not use –ve sign , we show the direction.
 Starting point will be taken as origin (x = 0) and the direction of particle movement is +x direction
Fx = max
mg – kv = max

k  F  kv  speed dependent so acceleration is var iable 


ax = g – v  
m  v  u  at , cannot apply 

Contact Us : Website : www.eduncle.com | Email : support@eduncle.com | Call Us : 7231923276 3.9


Mechanics

dv k
g v [Ist order differential equation]
dt m
dv k
 vg
dt m
dv
 mg – kv = m (use separation of variable to solve differential equation)
dt
mdv
dt 
mg  kv
t v
mdv
 dt  
0 0
mg  kv
v
mdv dx 1
t   ln  a  bx 
0
mg  kv a  bx b
m v
t  ln  mg  kv   0
k
kt
  ln  mg  kv   ln mg
m
kt  mg  kv 
  ln  
m  mg 
mg  kv kv
e kt / m  1
mg mg
kv
e kt / m  1  
mg

mg
v  1  e  kt / m 
k
v
mg
—–
k

O t
at t = 0 at t =  (e –
= 0)
mg
v = 0 v =
k

Example : A body of mass ‘m’ falls from rest at a height h under gravity (acceleration due to g) through a dense
medium which produces resistive force F = –kv2, where k is a conserve and ‘v’ is the speed. It will hit the ground
with a kinetic energy ?
Solution : Fx = max

Drag Force Always Applied opposite to the velocity.

mg – kv2 = ma

Contact Us : Website : www.eduncle.com | Email : support@eduncle.com | Call Us : 7231923276 3.10


Mechanics

(i) In Terms of Position (x)


x=0 u=0
dv
mg – kv = mv 2 h
dx 2
F = –kn
2
dv kv
v = g– +X
dx m
vdv mg
 dx
k x=h
g  v2
m
Integrating both sides
v x
v dv
0 k 2  0 dx
g v
m
k 2
Let g – v = t
m
2k
 v dv  dt
m
m
vdv = – dt
2k
k 2
g v
m
 m  dt
    x
g  2k  t
k 2
g v 2kx
log e  t   g m 
m
 k 2
g mv  2kx
log  
 g  m
 
2kx
kv2 
1 e m
mg
2kx
mg   
v2  1  e m 
k  
2kx
1 m2g   
K.E.  mv 2  1  e m 
2 2k  

x=0
(ii) In Terms of Time (t)
t=0 v=0
dv
mg – kv2 = m
dt
m dv x  dx 1  1 1  
dt    2    dx  kv2
mg  kv2  a x 2
2a  a  x a  x  

v
1  1 1 
   dx
2a  a  x a  x 

mg
dx 1
 a  bx  b ln  a  bx  +X
1
   ln  a  x   ln  a  x  
2a 
Contact Us : Website : www.eduncle.com | Email : support@eduncle.com | Call Us : 7231923276 3.11
Mechanics

dx 1 a  x 
a 2
 ln
 x 2 2a  a  x 
t v
dv
 dt  m  2 2
0 0  mg   kv 
v
 mg  kv 
m
t ln  
2 mg k  mg  kv  0

1 m   mg  kv  
t ln    0
2 gk   mg  kv  

kg  mg  kv 
2t  ln  
m  mg  kv 

mg  kv
e2 t kg / m

mg  kv
a c ab cd
  
b d ab cd

e2 t kg / m
1 2 kv
2 t kg / m

e 1 2 mg

mg  e2 t kg / m
 1
v  2t kg / m

k  e  1 

mg e t kg / m  e kg / m
 e t kg / m 
  t 
k et kg / m
 e
kg / m
e  t kg / m


mg  kg 
vt  tanh  t 
k  m

3.3 Impulse
When a force acts on an object for a short amount of time, impulse is the measure of how much the force changes
the momentum of an object. It is denoted by J or Jimp

Mathematical Forms
It can be represented as
(i) Force-time integral

J   Fdt

(ii) Product of the Average net force acting on an object and its duration
J = Faverage [t2 – t1]
J = Ft
(iii) Change in momentum of an object equals to the impulse applied to it. It is also known as Impulse momentum
Theorem.
J = P

Contact Us : Website : www.eduncle.com | Email : support@eduncle.com | Call Us : 7231923276 3.12


Mechanics

Units : SI unit of impulse is Newton-second


SI unit of momentum is kilogram meter per second.
These units of impulse and momentum are equivalent
N – s = kg.m/s

4. Motion of Bodies in Contact

(A) If two bodies with masses m1 and m2 are put in contact and if a force F is applied on the mass m1 then equal
acceleration is produced in both the bodies.

F m1
m2

Fig.
Acceleration produced in both bodies is given by,
F Force
a  ...(i)
(m1  m 2 ) total mass
Here F = force acting on the body with mass m1
If the force exerted by m2 on m1 is F’ Then
F – F’ = m1a
 F’ = F – m1a

F  m1 
 F  m1  F 1  
m1  m 2  m1  m 2 
m2 F
or, F'  m  m
1 2

(B) When the force acts on the body with mass m2. The acceleration product in the bodies

m1 F
m2

Fig.
F
a ...(i)
(m1  m 2 )
If the force exerted by m1 on m2 is F’ Then
F – F’ = m2a

m2 F  m2 
F'  F  m 2 a  F   F 1  
m1  m 2  m1  m 2 
(C) When three bodies are lying in contact on a horizontal smooth table.

m1 m3
m2

Fig. (A)
When the force acts on a body of mass m1

Contact Us : Website : www.eduncle.com | Email : support@eduncle.com | Call Us : 7231923276 3.13


Mechanics

F m1
m2 m3

Fig. (B)
Representation of action and reaction forces

F m1 T1 T1 m2 T2 T2 m3

Fig. (C)
The contact force between m1 and m2 is T1 and that between m2 and m3 is T2.
For the first body F – T1 = m1a ...(i)
For the second body T1 – T2 = m2a ...(ii)
For the third body T2 = m3a ...(iii)
Here, F = (m1 + m2 + m3)a ...(iv)
Solving these equation, we get,

F
a
m1  m 2  m3

(m 2  m3 ) F
T1 
m1  m 2  m3

m3 F
T2 
m1  m 2  m3
(D) If the force F acts on the body with mass m3
m1F
T1 
m1  m 2  m 3

m1 F
m2 m3

Fig.(D)
(m1  m 2 ) F
T2 
m1  m 2  m 3

Motion of Bodies Connected by Strings


(i) When two bodies of masses m1 and m2 are tied to the ends of a string and these are pulled by a force F.

Fig. (A)
T = m1a ...(i)
F – T = m2a ...(ii)

Contact Us : Website : www.eduncle.com | Email : support@eduncle.com | Call Us : 7231923276 3.14


Mechanics

Solving these equations,


F m1F
a and T 
m1  m 2 m1  m 2
Here, T = tension in the string
F = force acting on mass m2 in the forward direction.
(ii) If in the above example three bodies are pulled with a force F.
Acceleration produced in the system
m3
m1 T m 2 T T3
1 2
F

Fig.(B)
F
a
m1  m 2  m3
Tension is produced in the first string
m1F
T1  m1a 
m1  m 2  m 3
Tension in the second string
(m1  m 2 ) F
T2  ( F  T2  m 3 a)
m1  m 2  m3
Tension is the third string
T3 = F

Cases of Smooth Pulley


Case-1 : Let two bodies of masses m1 and m2 respectively are connected by a light in-extensible string passing over
a massless smooth pulley. If m1 > m2 then mass m1 moves downwards and mass m2 moves upwards.
Equation of motion
For body A, m1g – T = m1a
For body B, T – m2g = m2a
Solving these equations, we get

m1  m 2
a g T
m1  m 2
B
m2 T
2m1m 2
and T  m  m g a
1 2
m2g A
The reaction at the pulley m1
a
4m1m 2
R  2T  g m1 g
m1  m 2

Case-2 : Let us consider the case of a body of mass (m1) which is attached to a light string, rests on a smooth
horizontal plane. The String passes over a frictionless pulley fixed at the end of the plane. The other end of the string
carries a mass (m2) as shown in fig.

Contact Us : Website : www.eduncle.com | Email : support@eduncle.com | Call Us : 7231923276 3.15


Mechanics

T
m1

m2

m2g
Here we have
(m2g – T) = m2a ...(1)
and T = m1a ...(2)
Solving these equations we have
 m2   m1 .m 2 
a  g and T  m1a   g
 (m1  m 2 )   (m1  m 2 ) 

Case-3 : If in the above case, m1 placed on smooth inclined plane making an angle  with horizontal as show in fig. :

T
n
t io
mo T
m1 motion
m2


m2 g
In this case :
T – m1g sin  = m1a
and, (m2g – T) = m2a
Solving we get

(m 2  m1 sin ) g
a and T = m2g – m2a
(m1  m 2 )
 (m 2  m1 sin ) 
or, T  m 2 g 1  
 (m1  m 2 ) 
m1m 2 g
T (1  sin )
m1  m 2

Example : A ball of mass m1 and a block of mass m2 are attached by a lightweight cord that passes over a frictionless
pulley of negligible mass, as shown in Figure. The block lies on a frictionless incline of angle . Find the magnitude
of the acceleration of the two objects and the tensions in the cord.
y
y
n
T
a
T
m2 m2g sin 

 x
a m1 m1 x
m2g cos 
m1 g m2 g

(a) (b) (c)
Contact Us : Website : www.eduncle.com | Email : support@eduncle.com | Call Us : 7231923276 3.16
Mechanics

Solution : From free body diagram of the ball m1.


1. Fx = 0
2. Fy = T – m1g = m1ay = m1a
From the body diagram of the block m2.
3. Fx = m2g sin  – T = m2ax = m2a
4. Fy = n – m2g cos  = 0
In (3) we have replaced ax with a because that is the acceleration’s only component. In other words, the two
objects have accelerations of the same magnitude a, which is what we are trying to find. Equations (1) and
(4) provide no information regarding the acceleration. However, if we solve (2) for T and then substitute this
value for T into (3) and solve for a, we obtain
m 2 g sin   m1g
5. a
m1  m 2
When this value for ‘a’ is substituted into (2), we find
m1m 2 g  sin   1
6. T
m1  m 2

5. Spring-Mass System

Spring Potential Energy


When you compress or stretch a spring, as soon as the stress is relieved, the spring attains its normal shape instantly.
Its plastic potential energy helps it to do so generally, these elastic substances follow the Hooke’s law.

Hooke’s Law
According to the law, the force needed to change the shape of spring is
proportional to the displacement of spring. This displacement referred here is
now for the spring is compressed or stretched from its normal shape
Mathematically
F = – kx
where k = spring constant
work done by the spring is
xm
k( x m )2
W   Fdx   kx dx 
0
2
x m is displacement from the equilibrium position.
The work done by pulling force FP is
2
k(x m )
FP 
2
work done by the pulling force FP is positive as it has overcome the force of
spring.
Therefore
kxm2
W
2
In the process of displacement of the object from the initial displacement x i
to final displacement xf the work done is
xf
kx 2i kx 2f
W s    kx dx  
xi
2 2

This work done is in from of the spring potential energy.

Contact Us : Website : www.eduncle.com | Email : support@eduncle.com | Call Us : 7231923276 3.17


Mechanics

6. Projectile Motion

Projectile motion is the motion of an object thrown or projected into the air, subject to only the acceleration of the
gravity. The object is called a projectile and its path is called trajectory.

Assumption
 The free-fall acceleration is constant over the range of motion
(i) It is directed downward.
(ii) It is reasonable as long as the range is small compared to the radius of the Earth.
 The effect of air friction is negligible.
 With these assumptions, an object in projectile motion will follow a parabolic path and this path is called the
Trajectory.

Projectile Motion Diagram


y

uy = 0 v C g
vB
uy C
D uxi

vi uxi 
B
uy vD

uji
i E uxi
x
A uxi i

uy vE

Analysis
 Position at any time is given by
   1
rF  ri  vi t  gt 2
2
 Components of velocity in x and y direction
Vxi  Vi cos  and Vyi  Vi sin 
 The x-direction has constant velocity ax = 0
 y direction is free fall
ay = – g
Here we have two types of motions
y
1
– gt
2

2
vi t
(x, y )
r f+
x
O

Contact Us : Website : www.eduncle.com | Email : support@eduncle.com | Call Us : 7231923276 3.18


Mechanics

1. Motion of a particle under constant velocity is in the horizontal direction.


2. Motion of particle under constant acceleration is in the vertical direction.
Note : The y-component of the velocity is zero at the maximum height of the trajectory.

Range and Maximum Height of Projectile


y

A ui A – 0

vi

h
i B
x
O
R
Range R is the horizontal distance of the projectile
Maximum height the projectile reaches is h.

Vi2 sin 2 i
h
2g
This equation is only applicable for Symmetric Motion

Vi2 sin 2i


Range R 
g
This is also valid only for Symmetric Trajectory.

Relation between Range and Height of Projectile


R tan 
h
4
Note :
1. The Maximum Range occurs at i = 45°
2. Complementary angles will produce the same range.

7. Friction and Frictional Force

Sir Isaac Newton once wondered why apples on the trees fall to the ground. Trying to find an answer to this question,
he proposed the laws of gravity in 1687 and gave an answer to his own question, but if gravity acts downwards, why
do things which are pushed forwards on a surface eventually stop? Why do the wheels of a cart that is pushed forward
eventually stop? Why do we need to maintain a pushing force when we want to push something to a spot some distance
away? Attempts to answer such questions were made by Leonardo da Vinci as early as 1493 when he documented
the classic laws of sliding friction in his unpublished notebook. These laws were rediscovered in 1699 by Guillaume
Amontons. Friction is the force which resists motion between two surfaces which are sliding against each other. There
are various kinds of friction. And one among.
(A) Friction : The property by virtue of which an opposing force is generated between two rough surfaces in
contact with each other and which opposes the sliding of one surface over the other is known as friction.
(B) Frictional Force : The force which opposes the sliding or relative motion of two bodies in contact with each
other, is defined as the force of Friction or the Frictional force.

Contact Us : Website : www.eduncle.com | Email : support@eduncle.com | Call Us : 7231923276 3.19


Mechanics

7.1 Types of Frictional Force and Their Definition


Types of Frictional Force

Static Dynamic Sliding Rolling


Friction Friction Friction Friction

(i) Static Friction : The frictional force, which is effective before motion starts between two planes in contact
with each other, is known as static friction. In simple words when two bodies do not slip on each other, the
force of friction is called static friction.
(ii) Dynamic Friction : The frictional force, which is effective when two surfaces in contact with each other
are in relative motion with respect to each other, is defined as dynamic friction.
(iii) Sliding Friction : The frictional force, which is effective on sliding of two surfaces in contact with each
other, is known as the sliding friction.
(iv) Rolling Friction : The frictional force, which is effective when a body rolls or rotates on a surface, is defined
as rolling friction.
One more type of friction is fluid friction.
(v) Fluid friction : Air, water and oil are all fluids. Air resistance is a type of fluid Friction. As an object falls,
air resistance pushes up on the object. It is also called viscous force.
No Force
Motion
Static
friction
Sliding
Motion
Static
friction
Rolling Force
Motion
Rolling
friction
Is Air a Fluid ?
Yes, Air is made of stuff, air particles, that are loosely held together in a gas form.
Although liquids are the most commonly recognized fluids, gasses are also fluids.
Since air is a gas, it flows and takes the form of its container so it is a fluid.

7.2 Friction as the Component of Contact Force


When two bodies are kept in contact, electromagnetic forces act between the charged particles at the surfaces of the
bodies. As a result, each body exerts a contact force on the other. The magnitudes of the contact forces acting on
the two bodies are equal but their directions are opposite and hence the contact forces obey Newton's third law.
N = normal force
F = contact force

f = friction

Fig.
Contact Us : Website : www.eduncle.com | Email : support@eduncle.com | Call Us : 7231923276 3.20
Mechanics

The direction of the contact force acting on a particular body is not necessarily perpendicular to the contact surface.
We can resolve this contact force into two components, one perpendicular to the contact surface and the other parallel
to it (Fig.). The perpendicular component is called the normal contact force or normal force and the parallel component
is called friction.

7.3 Kinetic Friction


When two bodies in contact move with respect to each other, rubbing the surfaces in contact, the friction between
them is called kinetic friction. The directions of the frictional forces are such that the relative slipping is opposed
by the friction.
A
v
f
B
f
Fig.

Suppose a body A placed in contact with B is moved with respect to it as shown in figure (2). The force of friction
acting on A due to B will be opposite to the velocity of A with respect to B. In figure (2) this force is shown towards
left. The force of friction on B due to A is opposite to the velocity of B with respect to A. In figure (2) this force
is shown towards right. The force of kinetic friction opposes the relative motion. We can formulate the rules for finding
the direction and magnitude of kinetic friction as follows :
(a) Direction of Kinetic Friction
The kinetic friction on a body A slipping against another body B is opposite to the velocity of A with respect to B.
It should be carefully noted that the velocity coming into picture is with respect to the body applying the force of
friction.
A B
f
f v

Fig.

As another example, suppose we have a long box having wheels and moving on a horizontal road (figure 3). A small
block is placed on the box which slips on the box to fall from the rear end. As seen from the road, both the box
and the block are moving towards right, of course the velocity of the block is smaller than that of the box. What is
the direction of the kinetic friction acting on the block due to the box? The velocity of the block as seen from the
box is towards left. Thus, the friction on the block is towards right. The friction acting on the box due to the block
is towards left.

(b) Magnitude of the Kinetic Friction


The magnitude of the kinetic friction is proportional to the normal force acting between the two bodies. We can write
fk = k N ...(1)
where N is the normal force. The proportionality constant is called the coefficient of kinetic friction and its value
depends on the nature of the two surfaces in contact.
If the surfaces are smooth k will be small, if the surfaces are rough k will be large. It also depends on the materials
of the two bodies in contact.

Contact Us : Website : www.eduncle.com | Email : support@eduncle.com | Call Us : 7231923276 3.21


Mechanics

According to equation (1) the coefficient of kinetic friction does not depend on the speed of the sliding bodies.
Once the bodies slip on each other the frictional force is k N, whatever be the speed. This approximately true for
relative speeds not too large (say for speeds < 10 m/s).
We also see from equation (1) that as long as the normal force N is same, the frictional force is independent of
the area of the surface in contact. For example, if a rectangular slab is sliding over a table. The frictional force
is same whether the slab lies flat on the table or it stands on its face of smaller area (fig.)

f f

Fig.

7.4 Static Friction


Frictional forces can also act between two bodies which are in contact but are not sliding with respect to each other.
The friction in such cases is called static friction. For example, suppose several labourers are trying to push a heavy
almirah on the floor to take it out of a room (figure 4).

Fig.

The almirah is heavy and even the most sincere effort by them is not able to slide it on the floor even by a millimeter.
As the almirah is at rest the resultant force on the almirah should be zero. Thus, something is exerting a force on
the almirah in the opposite direction. In this case, it is the floor which exerts a frictional force on the almirah. The
labourers push the almirah towards left (in figure 4) and the floor exerts a frictional force on the almirah towards right.
This is an example of static friction.
Magnitude of static friction adjusts its value according to the applied force. As the applied force increases, the frictional
force also increases. The static friction is thus, self adjustable. It adjusts its magnitude (and direction) in such a way
that together with other forces applied on the body, it maintains ‘relative rest’ between the two surface. However, the
frictional force cannot go beyond a maximum. When the applied force exceeds this maximum, friction fails to increase
its value and slipping starts. The maximum static friction that a body can exert on the other body in contact with it,
is called limiting friction. This limiting friction is proportional to the normal contact force between the two bodies.
We can write
fmax = sN ...(1)
where fmax is the maximum possible force of static friction and N is the normal force. The constant of proportionality
is called the coefficient of static friction and its value again depends on the material and roughness of the two surfaces
in contact. In general, s is slightly greater than k. As long as the normal force is constant, the maximum possible
friction does not depend on the area of the surfaces in contact.

Contact Us : Website : www.eduncle.com | Email : support@eduncle.com | Call Us : 7231923276 3.22


Mechanics

Once again we emphasis that is s N is the maximum possible force of static friction that can act between the bodies.
The actual force of static friction may be smaller than sN and its value depends on other forces acting on the body.
The magnitude of frictional force is equal to that required to keep the body at relative rest. Thus,
f s  f max   s N ...(2)

7.5 Laws of Friction


We can summarise the laws of friction between two bodies in contact as follows :
1. If the bodies slip over each other, the force of friction is given by fk = kN
where N is the normal contact force and k is the coefficient of kinetic friction between the surface.
2. The direction of kinetic friction on a body is opposite to the velocity of this body with respect to the body
applying the force of friction.
3. If the bodies do not slip over each other, the force of friction is given by f s  s N
where s is the coefficient of static friction between the bodies and N is the normal force between them. The
direction and magnitude of static friction are such that the condition of no slipping between the bodies is
ensured.
4. The frictional force fk or fs does not depend on the area of contact as long as the normal force N is same.
Table gives a rough estimate of the values of coefficient of static friction between certain pairs of materials.
The actual value depends on the degree of smoothness and other environmental factors. For example, wood
may be prepared at various degrees of smoothness and the friction coefficient will vary.
Table : The Friction Coefficients
Material ms Material ms
Steel and steel 0.58 Copper and copper 1.6
Steel and brass 0.35 Teflon and teflon 0.04
Glass and glass 1.0 Rubber tyre on dry
1.0
Wood and wood 0.35 concreate road
Wood and metal 0.4 Rubber tyre on wet
0.7
Ice and ice 0.1 concrete road

Dust, impurities, surface oxidation etc. have a great role in determining the friction coefficient Suppose we take two
blocks of pure copper, clean them carefully to remove any oxide or dust layer at the surfaces, heat them to push out
any dissolved gases and keep them in contact with each other in an evacuated chamber at a very low pressure of
air. The blocks stick to each other and a large force is needed to slide one over the other. The friction coefficient
as defined above, becomes much larger than one. If a small amount of air is allowed to go into the chamber so that
some oxidation takes place at the surface, the friction coefficient reduces to usual values.

7.6 Rolling Friction


Rolling friction is the resistive force that slows down the motion of a rolling ball or wheel. It is also called Rolling
resistance.
When a force or torque is applied to a stationary wheel, there is a small static rolling friction force holding back the
rolling motion, however, resistance from static sliding friction is what really causes the wheel to start rolling.

Causes
1. The object is deformed at the point of contact with the surface.
2. The surface is deformed at the point of contact with the object.
3. Motion is created below the surface as a result of the above mentioned points.
Contact Us : Website : www.eduncle.com | Email : support@eduncle.com | Call Us : 7231923276 3.23
Mechanics

Coefficient of Rolling Friction


The general equation to calculate rolling friction is,
Fr = rN
where Fr is the rolling friction, or the resistance towards rolling objects,
r is the coefficient of rolling friction,
N is the normal reaction on the object
r, which is the coefficient of rolling frictions can be defined as the ratio of the force of the rolling friction to the
total weight of the object.
Rolling resistance can also be expressed as,
Fr = rW
where Fr is the rolling friction, or the resistance towards rolling objects,
r is the coefficient of rolling friction,
W is the weight of the object,

Block and Wedge with Friction


Friction
(a) For bodies not in relative motion
0  f  N
where N = reaction (Normal force)
f opposes the motion that showed occur in absence
(b) For bodies in relative motion
f = N, f opposes the relative velocity.
A block of mass 'm' rests on a fixed wedge of angle friction coefficient is . Find  at which the block starts to slide.

f
m y

f

x
w 
Solution : With the co-ordinate shown we have
w
sin
 

w
mx
  w sin   f

and, my
  N  w cos   0

When sliding starts, f has max value N and x = 0


w sin max  N 

w cos max  N 
Here, tan max = 

Contact Us : Website : www.eduncle.com | Email : support@eduncle.com | Call Us : 7231923276 3.24


Mechanics

Example : A solid cylinder C of mass 10 kg rolls without slipping in an inclined plane which offer friction. The angle
of the inclined plane with the horizontal is 30° as shown in the figure below. A massless inextensible string is wrapped
around the cylinder and it passes over a frictionless pulley, P. The other end of the string holds a block B of mass
2 kg as shown in the figure. Take g = 10 ms2. [IIT JAM Physics 2010]
(a) Draw the free body diagram of the cylinder and the block. If the block B moves down by 0.01 m, how much
distance does C move along the incline ?
(b) If the block B moves with an acceleration of 0.05g upward, find the magnitude of frictional force.

C B
30°
Solution :
(a) Free body diagram of cylinder
N
T

f
 
sin
g
M
Mg Mg cos
As in case of rolling, T
the lowest point of body is at rest.
Hence the loss of energy due to M
friction force.
  
 f  v dt  0 [ v  0] Mg
i.e. There is no loss of energy in this process. Hence if block moves down by 0.01 m, then the cylinder translational
displacement along the incline is only 0.01 m.
(b) From the free body diagram
T – mg = ma
or, T = m(a + g) = m(0.05g + g) = 1.05 mg
= 1.05 × 2 × 10 = 21
and for cylinder translational motion (of C.M.)
Mg sin – T – f = Ma
 f = Mg sin – T – Ma
1
 10  10  21  10  10  0.05  24N
2
= 24N

Contact Us : Website : www.eduncle.com | Email : support@eduncle.com | Call Us : 7231923276 3.25


Mechanics

Example : The coefficient of state is friction between the two blocks shown in figure, is m and the table is smooth.
What maximum horizontal force F can be applied to the block of mass M so that the blocks move together ?

M F

Solution : When the maximum force F is applied, both the blocks move together towards right. The only horizontal
force on the upper block of mass m is that due to the friction by the lower block of mass M. Hence this force on
m should be towards right. The force of friction on M by m should be towards left by Newton's third law. As we
are talking of the maximum possible force F that can be applied, the friction is limiting and hence f = N where N
is the normal force between the blocks.
Now consider the motion of m. The forces on m are :
(a) mg downward by the earth (gravity)
(b) N upward by the block M
N

f = N

mg
(c) f = N (friction) towards right by the block M.
In the vertical direction, there is no acceleration. This gives
N = mg ...(1)
In the horizontal direction, let the acceleration be a, then N = ma
or, mg = ma
a = g ...(2)
Next, consider the motion of M.
N N1

f = N F

mg

The forces on M are :


(a) Mg downward by earth (gravity)
(b) N1 upward by the table (normal force)
(c) N downward by m (normal force)
(d) f = N (friction towards left by m)
(e) F (applied force) by the experimenter
The equation of motion is
F – N = Ma
F –  mg = M m g [using (1) and (2)]
F = g (M + m)

Contact Us : Website : www.eduncle.com | Email : support@eduncle.com | Call Us : 7231923276 3.26


Mechanics

8. Uniform Circular Motion

The movement of a particle moving in a circle is called circular motion. If the particle is moving with constant
(uniform) speed is called Uniform Circular Motion. In this case, the speed is constant but velocity changes every
instant.
in this case
dv
0
dt

v v 2
a  lim 
t  0 t r
a is the magnitude of the acceleration. The direction of a is instantaneously along a radius inward or toward the centre
of the circle.
Here r is the radius of circle.
u
u Acceleration has
arad constant magnitude
arad but varying direction
u arad

arad u
Velocity and
a rad arad acceleration
are always
u
u perpendicular

Centripetal Acceleration and Centripetal Force


If a particle is moving in a circle, it must have some acceleration acting towards the centre which is making it to move
around the centre. Since this acceleration is perpendicular to the velocity of particle at every instant, only the direction
of velocity is changing and not magnitude and that's why the motion is uniform circular motion. We call this
acceleration centripetal acceleration (or radial acceleration), and the force acting towards the centre is called
centripetal force.
Now centripetal acceleration is
V2
ac   2 r
r
In case of non-uniform circular motion, there is some tangential acceleration due to which the speed of particle
increases or decreases.
dv
at 
dt
The resultant acceleration is the vector sum of radial acceleration and tangential acceleration. And centripetal force
is
mv 2
ma c 
r
This is not a special force, actually force like tension or friction may be a cause of origination of centripetal force.
Tangential acceleration arises due to change in speed (magnitude of speed), it may or may not be zero.
Radial acceleration arises due to change in direction.
When the vehicles turn on the roads, it is the frictional force between tyres and ground which provides the required
centripetal force for turning.
Contact Us : Website : www.eduncle.com | Email : support@eduncle.com | Call Us : 7231923276 3.27
Mechanics

So if a particle is moving in uniform circular motion:


1. Its speed is constant
2. Velocity is changing at every instant
3. There is no tangential acceleration
4. Radial (centripetal) acceleration = 2r
5. v = r

a
v
v a

Fig. : In uniform circular motion the acceleration a is


always directed towards the center of the circle
and hence is perpendicular to v.

Steps to Solve Problems Related to A Circular Motion


 Choose polar coordinate (Centre as Origin)
v2
for radially outward = +ve, ar = –
r
 r & perpendicular () to 
Resolve forces in  r direction
F sin 30 ^r

F
30°
30
cos
F
O

r and perpendicular to 
No need to resolve those forces, which are already in  r direction.
 Write radial Equation of motion at that instant at which we have to calculate some thing, in most cases except
when F (force) is function of time (t) or .
Fr  Ma r
F cos 30 = Radial force
F sin 30 = Tangential force
 Sometimes, we may have to write tangential equation
Ft  Ma t
 To calculate speed at some point, use conservation of energy. (If friction or drag force is not present) (that
we will discuss in unit 4)
If friction (drag force is present then speed is calculated using both radial and tangential equation).
 When motion or circle is horizontal then there is equilibrium in vertical direction.
Fup = Fdown

Contact Us : Website : www.eduncle.com | Email : support@eduncle.com | Call Us : 7231923276 3.28


Mechanics

Example : A hoop of radius a rotates with constant angular velocity  about the vertical axis as shown in the figure.
A bead of mass m can slide on the hoop without friction. if g < 2a, at what angle è apart from 0 and ð is the
bead stationary?
Solution : At a and b bead is at rest.
Bead is rotating with wire thus the motion of bead will be circular.
Let at c bead is at rest.

(a)

N
 N cos 

r N sin  (c) ^r
bead
(Rest)

(b) mg

Radial Equation of Motion


fr = mar
–N sin  = m(–2 r)
N sin2  = m2 r ...(1)
It is an horizontal motion thus
fup = fdown
N cos  = mg ...(2)
r
sin  =
a
r = a sin 
by (1) and (2)
N sin  = m2 a sin 
by (2) – N cos  = mg ...(3)
by (1) – N = m2a ...(4)
(3) ÷ (4)
g
cos  =
2 a

Contact Us : Website : www.eduncle.com | Email : support@eduncle.com | Call Us : 7231923276 3.29


Mechanics

Example : A particle slides on the inside surface of a frictionless cone. The cone is fixed with its tip on the ground
and its axis vertical, as shown in the figure on the right. The Semi-Vertex angle of the cone is . If the particle moves
in a circle of radius r0, without slipping downwards, the angular frequency  of this motion will be
N sin 

^n

 ^r
O r0
n cos 
mg

Solution : Radial EOM


Fr = mar
–N cos  = m(–2r0)  angular acceleration
N cos  = m2r0 ...(1)
motion is horizontal Fup = Fdown
N sin  = mg ...(2)
by (2) ÷ (1)
g
tan  =
2 r0
g
2 =
r0 tan 
g
 =
r0 tan 

Example : A bike stuntman rides inside a well of frictionless surface given by z = (x2 + y2), under the action of gravity

acting in the –ve z direction g   gzˆ . What speed should he maintain to be able to ride a constant height z0 without
falling down
Solution : Z = a(x2 + y2)  parabola in both direction x and y
z

^r

x (x, 0, z 0)

Paraboloid z0 mg
(Revolution of parabola)

consider the moment when bike is in x-z plane, at height z = z0


(x, 0, z0)
Contact Us : Website : www.eduncle.com | Email : support@eduncle.com | Call Us : 7231923276 3.30
Mechanics

Radial EOM
Fr = mar

  v2 
–N cos  = m   ,  is not mention in the Ques and speed is asked; So we use this relation
 r 
mv 2
N cos  = ...(1)
r
 circle is Horizontal thus
Fup = Fdown
N sin  = mg ...(2)
by (2) ÷ (1)
rg
tan  =
v2
rg
v = (Here radius is x)
tan 
xg
v =
tan 
given, z = a(x2 + 0)
z = ax2
dz
tan (90 – ) =
dx
z Perpendicular

z
N 90 – 
90°
 ^ 90 – 
r
x

mg
90 – 
x
x

dz
tan (90 – ) =
dx
cot  = 2ax

v  xg co t 

v  xg2ax

v  2gax 2  2gz  2gz 0

Contact Us : Website : www.eduncle.com | Email : support@eduncle.com | Call Us : 7231923276 3.31


Mechanics

9. Frame of Reference

We have studied about position, velocity, acceleration and distance/displacement, but all these quantities require a frame
of reference from which they are measured.
So, basically reference frame or frame of reference is used in dynamics, are known as coordinate system with axes
emanating from a point known as the origins.
Example : Imagine you threw and caught a ball while you were on a train moving at a constant velocity past station
to you, the ball will simply go vertically up and then down under the influence of gravity, however, if there’s an
observer on the station platform, then that person will see the ball travel in a parabola, with a constant horizontal
component of velocity equal to the velocity of the train.

Observer on train Observer at station

Example :
V
A B

If we ask A what velocity of B is, he will say it is at the rest, but if we ask same question to C, he will say B is
moving with a velocity V in the positive x direction.
So we obtained different observation with respect to different persons, that means we can see before specifying the
velocity we have to specify in which frame we are.

Frames of reference and relative motion is actually the reason that people
get car sick. Your brain is getting two different sets of information about your
body’s motion that might not exactly agree with each other; information from
Di d Yo u your eyes, and information from your inner ear. Some people are more
Kn o w ? sensitive to these differences, which causes them to feel car sick as they
watch the road “whiz” by. If you are prone to getting car sickness, try to look
forward at a point far in the distance and stay focused on that.

Simulation from Frame of Reference


(i) Reference Frame is Rest

The acceleration of the mass will be, say a rest

Therefore the force on it will be Frest
We will reason that
 
Frest  ma rest

(ii) Reference frame Starts moving with Constant Velocity Vector v



The acceleration of frame = a  0

Contact Us : Website : www.eduncle.com | Email : support@eduncle.com | Call Us : 7231923276 3.32


Mechanics

Thus, acceleration of mass m relative to frame is given by


   
a inertial  a rest  a  a rest

Force on it will be Fint ertial and we will reason that
   
Fint ertial  ma inertial  ma rest  Frest
y y’

O O’ x x’

z z’

(iii) Reference Frame moves with Constant Acceleration



Let the acceleration of frame be a frame .

Thus, acceleration of mass related to frame will be a rel
    
a rel  a in ertial  a frame  a rest  ma frame

Let there be force F
   
Frest  ma rel  ma rest  ma frame
   
 Frest  m( a frame )  f rest  Fpseudo
in this case we obtained that some pseudo forces are acting on it. On the basis of that we can divide frame of
references in two parts.

9.1 Types of Frame of References


There are two types of frame of references :
1. Inertial
2. Non-Inertial

Inertial frame of References


Inertia is the property of matter in which an object that is at rest wants to remain at rest, and an object that is moving
wants to remain moving in a straight line unless another force acts upon it, i.e. it is in a uniform motion.
Likewise, an inertial frame of reference is a reference frame in which an object stays either at rest or at constant
velocity unless another force acts upon it.
“An inertial frame of reference in classical physics and special relatively possesses the property that in this frame of
reference a body with zero net force acting upon it does not accelerate”.
 In analytical terms, it is defined as a frame of reference that describes time and space homogeneously,
isotropically and in a time independent manner.
 Conceptually, the physics of a system in an inertial frame have no cause external to system.
 “All the newton’s laws are valid in this frame of reference”.
 The term inertial frame is relative.
 An inertial frame of reference may also be called an inertial reference frame, inertial frame, reference frame
or inertial space.

Contact Us : Website : www.eduncle.com | Email : support@eduncle.com | Call Us : 7231923276 3.33


Mechanics

An inertial frame is endowed with the following characteristics :


 All the fundamental laws of physics are valid in inertial frames.
 All the fundamental laws of physics assumes the same mathematical
shape in all inertial frame.
 Inertial frames are isotopic with respect to mechanical and optical
experiments
 The optical experiments performed in an inertial frame in any direction will
always yield the same results.

Non-Inertial Frame of Reference


 non-inertial frame as a frame which is accelerated with respect to assumed inertial frame of reference.
 Newton’s laws does not hold true in these frames.

Example :
1. A rotating merry-go-round
2. A turning car with constant speed.
3. The Rotating earth
 If we want to make newton’s law hold here we need to take account some force, known as pseudo
forces.
 A rotating frame of reference is a non-inertial frame of reference.

Is earth an inertial frame of reference?


Earth rotates around its axis as also revolves around the sun. In both these
motion, centripetal acceleration is present. Therefore, strictly speaking earth or
any frame of reference fixed on earth cannot be taken as an inertial frame.
However, as we are dealing with speeds (speed of light) and speed of earth is
only about, therefore when small time intervals are involved, effect of rotation and
revolution of earth can be ignored. Furthermore, this speed of earth can be
assumed to be constant. Hence earth or any other frame of reference set up on
earth can be taken as an approximately inertial frame of reference.
On the contrary, a frame of reference which is accelerated or decelerated is a
non-inertial frame.
Other examples of inertial frames of reference are :
(i) A frame of reference remaining fixed w.r.t. stars.
(ii) A space ship moving in outer space, without spinning and with its engine
cut off.
Z

North Pole
(X, Y, Z)

X Y

Contact Us : Website : www.eduncle.com | Email : support@eduncle.com | Call Us : 7231923276 3.34


Mechanics

Example :
elevator elevator up elevator down
stopped (a=0) (a=g) (a=–g)

–mg

mg mg

W = mg W = 2mg W=0

9.2 Simple Pendulum in Linearly Accelerated Frame


We will discus this topic in detail in unit 5.
1. Horizontally Accelerated Frame
The equilibrium position of pendulum is as shown in figure :

a0

ma 0

mg mg

For equilibrium, we must have

T cos

ma0
T sin

mg

T sin = ma0 ...(1)


T cos = mg ...(2)
a0
tan  
g

a 
  tan 1  0 
g 
 is the angle made by the string with horizontal.

Contact Us : Website : www.eduncle.com | Email : support@eduncle.com | Call Us : 7231923276 3.35


Mechanics

Time period of oscillation :


Time Period  2
g

a0

ma 0

Equilibrium
Position

Note : If any force passes through line of suspension then it does not have any effect.

mg


T  2
g (eff )

geff = Resultant of a0 and g

g eff  a 0 2  g 2


Hence, T  2
a 0  g2
2

Lift Accelerating Upwards

a0

mg
ma 0

geff = g + a0


T  2
a0  g

Contact Us : Website : www.eduncle.com | Email : support@eduncle.com | Call Us : 7231923276 3.36


Mechanics

Lift Accelerating Downwards

a 0>g
mg

geff = g – a0


T  2
g  a0

Suspension moving at an Angle  with Horizontal

a0

 (m)
–
90
ma0 mg

g eff  a 02  g 2  2a 0 g cos(90  )

 a 02  g 2  2a 0 g sin 


T  2
g eff

Example : A simple pendulum is suspended from point of a large box (cube) which is sliding on a smooth inclined
plane of inclination ‘’. What is the time period of simple pendulum.
Solution :

sin
g

The large box (cube) is sliding with acceleration g sin .


Considering pendulum 

g sin 

g cos 
g

Contact Us : Website : www.eduncle.com | Email : support@eduncle.com | Call Us : 7231923276 3.37


Mechanics

2
Here, g eff  g 2   g sin    2gg sin  cos  90   

g eff  g 2  g 2 sin 2   2g 2 sin 2 

g eff  g 2  g 2 sin 2   g cos 


Hence, T  2
g eff


T  2
g cos 

10. Uniformly Rotating Frame

It is a type of non-inertial reference frame, that is rotation related to an inertial reference frame.
Eg. : Surface of the earth
Uniformly rotation frame is characterized by two forces
1. Centrifugal force
2. Coriolis force
Suppose that a frame S’(Xr, Yr, Zr) is rotating with an angular velocity related to an inertial frame S(Xi, Yi, Zi). For
simplicity, we assume that both of the frames have common origin O and common Z-axis.
In case of the Earth, the common origin O may be considered as the centre of the earth, Z-axis as coinciding with
its rotational axis and the frame S’ is rotating with earth relative to the non-rotating frame S.
The position vector of a particle P in both frames will be the same, i.e., Ri = Rr = R, because the origins are coincident.
Now, if the particle P is stationary in the frame S, the observer in the rotating frame S will see that the particle is
 dR 
moving oppositely with linear velocity – × R. Thus, if the velocity of the particle in the frame S is   , then
 dt i
 dR 
its velocity   in the rotating frame will be given by
 dt  r
 dR   dR   dR   dR 
       R or       R ...(1)
 dt  r  dt i  dt i  dt  r
Zi = Zr
S S’ P
=R


r
=R

Yr
i
R

Yi
O

Xi
Xr
Fig.
d d
     ...(2)
 dt i  dt  r
vi = vr +  × R ...(3)

Contact Us : Website : www.eduncle.com | Email : support@eduncle.com | Call Us : 7231923276 3.38


Mechanics

 dvi   dvi 
 dt    dt     v i
 i  r
 dv i   d 
 dt    dt (v r    R)     (v r    R)
 i  r
d
ai = ar + 2  vr +  × ( × R) + × R
dt
d
For Earth,  is constant, So  0 . Then
dt
ai = ar + 2  vr +  × ( × R) ...(4)
 ma r  ma i  2m  v r  m  (  R) 
                 
 This term is called This term is called 
 coriolis force first fictions force 

Fcentrifugal   m  (  r)
Fcoriolis  2m(  v)

11. Fictitious or Pseudo Forces

A pseudo force is an apparent force that acts on all masses whose motion is described using a non-inertial frame of
reference, such as rotating reference frame..
Pseudo force introduced when the frame of reference has started acceleration compared to a non accelerating frame.
Eg : centrifugal force , Coriolis force, Euler force
A force, which does not really act on the particle but appears due to the acceleration of the frame, is called a fictitious
or pseudo force. Hence fictitious force on the particle P is
F0 = –ma0 ...(1)
Here, the accelerated frame S’ is non-inertial.
Now, if a force Fi is applied on the particle and ai is the observed acceleration in the inertial frame (S), then according
to Newton’s law
Fi = mai ...(2)
Suppose frame S’ coincides at t = 0 with the inertial frame S. Then at any time t, the position vectors of a particle
ri and rn in the inertial and non-inertial frames respectively are connected as
1
ri  rn  a 0 t 2
2
where a0 is the acceleration of the frame S’ with respect to S. Double differentiation with respect to time t gives

d 2 ri d 2 rn
 2  a0 ...(3)
dt 2 dt
d 2 ri d 2 rn
As 2 = ai is the acceleration in the inertial frame, and  a n , is the acceleration observed in the non-inertial
dt dt 2
frame, we can write eq. (3) as
ai = an + a0 ...(4)
or, ai – a0 = an
or, mai – ma0 = man ...(5)
Fn = Fi + F0 ...(6)

Contact Us : Website : www.eduncle.com | Email : support@eduncle.com | Call Us : 7231923276 3.39


Mechanics

where Fi (= mai) is the real force acting on the particle and F0(= –ma0) is the fictitious force. Thus, the observer in
the accelerated frame will measure the resultant (total) force which is the sum of real and fictitious forces on the
particle i.e.,

Total force = True force + Fictitious force

11.1 Centrifugal Force


If an object is moving in a circle and experiences an outward force than this force is called centrifugal force.
Centrifugal force is given by :

mv 2
FC     m2 r
r
Fi = – m2r ...(7)
Fi = T = – m2r ...(8)
Y
Y’
S’ S X’

r Centrifugal
force
T
X
O

Centrifugal force
Fig.

Fn = man = 0 ...(9)
Now Fi + F0 = Fn ...(10)
i.e., – m2r + F0 = 0
Thus F0 = m2 r ...(11)
T + F0 = 0
This means that in the non-inertial frame the centrifugal force is balanced by the inward tension in the string. In
general, in the rotating frame, the centrifugal force is equal and opposite to the actual force and both are acting on
the same particle. Remember that the centrifugal force is a pseudo force and appears in the rotating frame due to its
rotation and it is not to be confused with the reactionary force acting on the centre away from it (on the hand in the
stone-string arrangement).

11.2 Coriolis Forces


Coriolis force is an inertial or fictitious force, that acts on objects that are in motion within a frame of reference, that
rotates with respect to inertial frame.
Deflection of an object due to the Coriolis force is called the Coriolis effect. In a reference frame with clockwise
rotation the force acts to the left of the motion of the object, and for anticlockwise motion force acts to the right.
(– 2m × vr) is a fictitious force which acts on a particle only if it is moving with respect to the rotating frame. Thus,
in the rotating frame if a particle moves with velocity vr, then it always experiences a force (– 2m × vr) perpendicular
to its path opposite to the direction of vector product  × vr. Due to the Coriolis force a moving particle in the northern

Contact Us : Website : www.eduncle.com | Email : support@eduncle.com | Call Us : 7231923276 3.40


Mechanics

hemisphere is deflected towards the right of its path. In the southern hemisphere, the deflection is towards the left
of the path. The effect of the Coriolis force is appreciable when it acts horizontally or has a horizontal component
because in the vertical direction its effect is retarded by the large gravitational force.
N

high >0

E low W

S
FC  2m( v r )

Pseud Forces on Earth


2 2
   7.25  105 rad / sec
T 24 hour

North Pole

N
E
W
S
Equator
(biggest latitude) N
W F
S

South Pole
Longitude




r

r

R

Equatorial plane

r = R cos 
Fcf = m2r

Fcf  m2 R cos 

Here, R is radius vector of earth and  is angle making by radius vector with equatorial plane.
Contact Us : Website : www.eduncle.com | Email : support@eduncle.com | Call Us : 7231923276 3.41
Mechanics

Earth as Rotating Frame


Vertically Up

N
W
E
S

 East, West, North and South directions on Earth surface are specified locally
 We use following coordinate system on Earth surface
 E  +X, W  –X
 N  +Y, S  –Y
 Vertically Up  +Z,
 Vertically Down  –Z
  lies in the yz-plane –

+z
90

y


90 – 
z 90
–

For Northern Hemisphere :



    sin   kˆ    cos   ˆj
For Southern Hemisphere :

     sin   kˆ    cos   ˆj

Example : A Person of mass 'm' is moving from East to west with speed 0 on Earth surface at a plane where latitude
is . Calculate apparent weight of person by considering centrifugal and Coriolis force on the person.
Solution : Apparent weight = Normal Reaction
  
Fcor  2m   r  

 2m  v0 ˆi  sin kˆ  cos ˆj 


   
 
 2m 0  sin ˆj  cos kˆ 

Contact Us : Website : www.eduncle.com | Email : support@eduncle.com | Call Us : 7231923276 3.42


Mechanics

Also, Fcf = m2 r


Fcf = m2 R cos 
Hence for vertical equilibrium
N + Fcf cos  = mg + 2m0 cos 
N + m2 cos2  = mg + 2m0 cos 
N = mg + 2mv0 cos  – m2 cos2 

Free Fall of a Body on Earth's Surface


Earth is a rotating system with  = 0.7 × 10–4 rad/sec. Take latitude . Earth rotates about its own diameter axis with
angular velocity .

N 

)
ge
pa
y’(east)

e
th
P

e
si d
n 
si

(in

 cos  
x’(south)
O

Make P is the falling point of the ball at lattice .


Taking x' axis southwards, y' eastwards and z' axis along OP (upwards). Neglecting the variation of 'g' with height.
The acceleration in body (earth) axes.
   
a '  g  2    '
   
Equation of motion = ma '  mg  2m     '

 '  ˆi ' x '  ˆj' y '  kˆ 'z ';   ˆi ' x  ˆj'  j  kˆ ' z
 
x = – cos , y = 0, z =  sin 

ma  g '  2m  y sin ˆi  ˆj   x sin    z cos    kˆ  y cos 
 
Then equation of motions of free fall
d2 x ' dy '
m 2 = 2m ' sin  = 2m sin 
dt dt
d2 y '  dx ' dz ' 
m 2
 2m  sin   cos  
dt  dt dt 
d2 z ' dy '
m 2 = –mg + 2m cos 
dt dt
Now assuming that the deviation is small. So, as velocity in x' & y' direction.
dy ' dx '
 0
dt dt
d2 y ' dz '
 m 2 = –2 cos m
dt dt
d2 z ' dz ' dz '
m 2 = –mg ; = –gt + c (c = 0 as at t = 0, = 0)
dt dt dt
Contact Us : Website : www.eduncle.com | Email : support@eduncle.com | Call Us : 7231923276 3.43
Mechanics

d2 y '
Then, = 2 cosgt
dt 2
dy ' t2
or, = 2 cosg + c1
dt 2
dy '
At t = 0, = 0; c1 = 0
dt
dy ' t3
=  cos gt2 ; y' =  cos g + c2
dt 3
At t = 0, y' = 0 ; c2 = 0
1
Hence, y' = gt3cos 
3
2h
If 'h' is the height of free fall t = g
3 1
1  2h  2 1  8h 3  2
We get, y' = g   cos      cos 
3  g  3  g 

Example : Calculate the fictitious force and the observed (or total) force on a body of mass 5 kg in a frame of
reference moving (i) vertically upwards, (ii) vertically downwards, with an acceleration of 4m/s2. (Take g = 9.8 m/s2).
Solution : Taking the earth to be an inertial frame of reference and the upward direction as positive, we have
Case (I) : True weight of the body or the true force acting on it, i.e., F = mg = 5(–9.8) = –49.0 newton, i.e., 49.0N
in the downward direction.
And, the fictitious force acting on it, i.e., F0 = mass (reversed acceleration of the non-inertial frame) = m(–a0)
= 5(–4) = –20 newton, i.e., 20N in the downward direction.
  
 observed or total force on the body, F  F  F0 = 49.0 + 20.0 = 69.0N in the downward direction, (i.e., the same
as in a life or elevator moving upwards with an acceleration of 4 m/s2)
Thus, the fictitious force in the body is 20 N and the observed (or total) force on it, 69 N, both acting downwards.

Case (II) : True force acting on the body, as before, F = mg = 5(–9.8) = –49.0 newton = 49.0N downwards.

And, the fictitious force acting on the body, F0 = m(–a0) = 5[–(–4)] = 20.0 N upwards.

 observed or total force on the body.


  
F  F  F0 = –49.0 + 20.0
= –29.0 newton
= 29.0 N downwards,
(i.e., the same as in a lift or elevator, moving downwards with an acceleration of 4 m/s2).
Thus, in this case, fictitious force on the body is 20N upwards and the observed (or total) force on it, 29 N downwards.

Example : A bullet is fired horizontally with a velocity 500 ms–1 at 60°N colatitude, (a) Eastwards (b) Northwards.
Calculate the magnitude of the Coriolis acceleration due to rotation of earth. [P.U. 2001]

Solution : Velocity of the bullet v = 500 ms–1
Co-latitude  = 60°
1
 Latitude  = (90 – ) = 30° and sin  = sin 30 = .
2

Contact Us : Website : www.eduncle.com | Email : support@eduncle.com | Call Us : 7231923276 3.44


Mechanics

The X-axis is taken towards the East, Y-axis towards the North and Z-axis in the vertically upwards direction.
Coriolis acceleration is given by
  
a a  2   v a

Horizontal component of Coriolis acceleration (a a ) H = 2 sin  (v y ˆi  v x ˆj) and Vertical component of Coriolis acceleration

(a a ) V  2 v x cos kˆ

(a) Bullet fired Eastward



The vx = v = 500 ms–1; vy = 0
 Horizontal component of Coriolis acceleration.

 a a H  2 sin v ˆj
2 1
= 2 ×  × 500 = 0.036 ms–1
24  60  60 2
along  ĵ i.e., –Y direction or towards South.
Vertical component of Coriolis acceleration

 a a V  2 v x cos kˆ
2 3
 2   500  0.036 3
24  60  60 2
along + k̂ i.e., Z direction or vertically upwards.
 Magnitude of the resultant acceleration
  2  2 2 2
a a   a a H   a a V   0.036   0.036 3
   0.072 ms 1

(b) Bullet Fired Northwards



Then vy = v = 500 ms–1; vx = 0

 Horizontal component of coriolis acceleration  a a H  2 sin viˆ = 0.036 ms–1 along î i.e., +X direction.

Vertical component of coriolis acceleration  a a V  2v x cos kˆ = 0
 Resultant acceleration = 0.036 ms–1 along +X direction.

Example : Calculate the rate of rotation of the plane of oscillation of a pendulum at latitude 30° and hence obtain the
time it will take to turn through (i) a full right angle and (ii) 60°. [P.U., 2001]
1
Solution : At a latitude  = 30°; sin  =
2
2 2
 The period of rotation T =  = 24 × 2 = 48 hours.
 sin   1
2
2
Hence rate of rotation = radian/hour
48
/2 48
and time taken to turn through (i) a full right angle or /2 radian =  = 12 hours
2 / 48 4
/3 48
and (ii) through 60° =  = 8 hours.
2 / 48 6
Contact Us : Website : www.eduncle.com | Email : support@eduncle.com | Call Us : 7231923276 3.45
Mechanics

Example : A circular disc (in the horizontal xy-plane) is spinning about a vertical axis through its center O with a

constant angular velocity  . A particle is observed to execute uniform circular motion, in the anticlockwise sense,
centered at P. When the particle is at the point Q, which of the following figure correctly represents the directions
 
of the Coriolis force Fr and the centrifugal force Fck [IIT JAM Physics 2008]

  
Fwr,Fck P Fw r,  P
Q Q Fck
(A) O (B) O
 

 
 Fwr  Fwr
Fck Q P Fck Q P

(C) O (D) O
 

Solution :


r
p
Q
v
o
y
z

x’ x
p

z’ y’


  
Fcor  2m    v 

 2m   kˆ  ˆj 
  
 
 2m ˆi

Fcent   m   r 

 m   kˆ  r ˆj 

 

  m  kˆ  r  ˆj 
 
  m2 r ˆi

Contact Us : Website : www.eduncle.com | Email : support@eduncle.com | Call Us : 7231923276 3.46


Mechanics


Example : Consider an object moving with a v velocity in a frame which rotates with a constant angular velocity

 . The Coriolis force experienced by the object is [IIT JAM Physics 2019]
 
(A) along v (B) along 
 
(C) perpendicular to both v and  (D) always directed towards the axis of rotation
Solution : (C) Coriolis force is given by
= –2m × v = –2m × v
Thus coriolis force is proportional to a cross product of two vectors, so it is perpendicular to both vector.
Thus statement C is correct.

Example : A person is standing at the edge of a disc of radius R. The disc is rotating about its axis with uniform
R
angular velocity . The person throws a stone in radially outward direction with speed relative to the disc.
2
Calculate acceleration of stone as seen by the person soon after throwing. (Neglect gravity).
Solution : As seen by the person the stone is acted upon by centrifugal and coriolis forces. This direction of the two
forces soon after throwing is shown in the figure.
Net force on the stone is

’
Fcf
90°
Fco r
2
F  Fcor  Fcf2
2
   R  2
 (2m  ) 2  (m2 r ) 2   2m. . sin 90    m2 R 
 2 
F  2m2 R
Therefore acceleration of stone as seen by the person is
F
a   2 2 R
m

Example : A particle of mass m moves radially outward with a constant velocity 0, on a turn table rotating with
uniform angular velocity . Find coefficients of friction.
Solution : On the turn table particle experiences coriolis and centrifugal forces.

v0 2
m r

2mv 0

Contact Us : Website : www.eduncle.com | Email : support@eduncle.com | Call Us : 7231923276 3.47


Mechanics

  
Fcoriolis  2m    0
 2m   0 zˆ  rˆ

 2m   0 ˆ
   
Fcentrifugal   m   (  r)

  m  ( r ˆ )
 m 2 rˆ
Net force on the particle

F= |Fcoriolis |2 +|Fcentrifugal |2
2
2 2 2 2 4 2  r 
F  4m    m  r 0  F  m 0 4 
 0 
Since, 0 is constant, the net force is balanced by friction force.
 fr = F
2
 r 
mg  m  0 4 
 0 
2
 0  r 
  4 
g  0 

The Period of Precession of Foucault’s Pendulum


2
T
 sin 
The earth rotates once in 24 hours with an angular velocity , hence
2 24 hours
24 hours  or, T 
 sin 
Thus, the rotation of the plane of oscillation of Foucault’s pendulum demonstrates the fact that the earth rotates about
its axis.

Contact Us : Website : www.eduncle.com | Email : support@eduncle.com | Call Us : 7231923276 3.48


Physics (MSP)

Physics (PH)
Model Solved Paper

Duration : 180 minutes Maximum Marks : 100

Read the following instructions carefully.

1. This test paper has a total of 60 questions carrying 100 marks. The entire question paper is divided
into Three Sections A, B and C. All sections are compulsory. Questions in each section are of
different types.

2. Section – A contains Multiple Choice Questions (MCQ). Each MCQ type question has four
choices out of which only one choice is the correct answer. This section has 30 Questions and carry
a total of 50 marks. Q.1 – Q.10 carry 1 mark each and Questions Q.11 – Q.30 carry 2 marks each.

3. Section – B contains Multiple Select Questions (MSQ). Each MSQ type question is similar to
MCQ but with a difference that there may be one or more than one choice(s) that are correct out
of the four given choices. The candidate gets full credit if he/she selects all the correct choices only
and no wrong choices. This section has 10 Questions and carry 2 marks each with a total of 20
marks.

4. Section – C contains Numerical Answer Type Questions (NAT). For these NAT type questions,
the answer is a real number which needs to be entered using the virtual numerical keypad on the
monitor. No choices will be shown for these type of questions. This section has 20 Questions and
carry a total of 30 marks. Q.1 – Q.10 carry 1 mark each and Questions Q.11 – Q.20 carry 2 marks
each.

5. In all sections, questions not attempted will result in zero mark. In Section – A (MCQ), wrong
answer will result in NEGATIVE marks. For all 1 mark questions, 1/3 marks will be deducted for
each wrong answer. For all 2 marks questions, 2/3 marks will be deducted for each wrong answer.
In Section – B (MSQ), there is NO NEGATIVE and NO PARTIAL marking provisions.There is NO
NEGATIVE marking in Section – C (NAT) as well.

Contact Us : Website : www.eduncle.com | Email : support@eduncle.com | Call Us : 7231923276 1


Physics (MSP)

Section-(A) Multiple Choice Questions (MCQ)

1. A sphere of mass M and radius r slips on a rough horizontal plane. At some instant, it has translational velocity
Vo
Vo and rotational velocity about the centre . Find the translational velocity after the sphere starts pure rollings ?
2r
6 5
(A) V (B) V
7 o 7 o
4 2
(C) V (D) V
7 o 7 o

2. A point charge ‘q’ is situated at a distance ‘d’ from the centre of a grounded conducting sphere of radius ‘R’ (d > R).
Value of the image charge is q at a distance ‘b’ from the centre. The quantities q and ‘b’ will be respectively :

R R2  R2  d2
(A)    q and (B)    q and
d d  d  R

R R2 R d2
(C)    q and (D)    q and
d d d R

3. A current time curve is shown in the following diagrams. This type of current is passed in the primary coil of
a transformer. The nature of induced e.m.f. in the secondary coil will be :

Ip Ip

(A) t (B) t

Ip t Ip t
(C) (D)

4. A particle of mass m whose total energy is twice its rest energy collides with an identical particle at rest. If they
stick together what is the mass of the resulting composite particle and what is its velocity ?
c c
(A) (B)
2 4
c
(C) (D) None of these
3

Contact Us : Website : www.eduncle.com | Email : support@eduncle.com | Call Us : 7231923276 2


Physics (MSP)

5. EFGH is a thin square plate of uniform density  and side 4a. Four point masses, each of mass m, are placed on
the plate as shown in the figure. In the moment of inertia matrix I of the composite system,
y
E F
m m
(–a,a,0) (a,a,0)
x
m (a,–a,0)
(–a,–a,0) m

H G
(A) Only Ixy is zero (B) Only Ixz and Iyz are zero
(C) All the product of inertia terms are zero (D) None of the product of inertia terms is zero

6. A space crew has a life support system that can last only for 1000 hours. What minimum speed would be required
for safe travel of the crew between two space stations separated by a fixed distance of 1.08 × 1012 km ?
c c
(A) (B)
3 2
c c
(C) (D)
2 5

7. A binary system consists of two stars of equal mass m orbiting each other in a circular orbit under the influence
of gravitational forces. The period of the orbit is T. At t = 0, the motion is stopped and the stars are allowed to fall
towards each other. After what time t, expressed in terms of T, do they collide?

x 2 dx x   x 
    x 2  sin 1  
x 2 2 2  

(A) 2 (B)
2
 
(C) (D)
2 2 4 2

8. A plane electromagnetic wave travelling in vacuum is incident normally on a non-magnetic, non-absorbing medium
of refractive index n. The incident (Ei), reflected (Er) and transmitted (Et) electric fields are given as
(Ei = E0i exp [i (kiz – t)],
Er = E0r exp [i (krz – t)];
Et = E0t exp [i (ktz – t)]
If E0i = 2 V/m and n = 1.5 then the application of appropriate boundary conditions leads to—
3 7 1 9
(A) E 0r   V / m, E 0t  V / m (B) E 0r   V / m, E 0 t  V / m
5 5 5 5
2 8 4 6
(C) E 0r   V / m, E 0t  V / m (D) E 0r  V / m, E 0t  V / m
5 5 5 5

Contact Us : Website : www.eduncle.com | Email : support@eduncle.com | Call Us : 7231923276 3


Physics (MSP)

9. The group velocity of electromagnetic waves moving with phase velocity c in a dispersive medium of refractive
index n is given by—
c c
(A) (B)
 dn   d 
  n d    n dn 

c c
(C) (D)
 dn   d 
 n   d   n   dn 

1 a
10. A particle of mass m is moving in a potential V(x) = m02 x 2  where 0 and a are positive constants.
2 2mx 2
The angular frequency of small oscillations for the simple harmonic motion of the particle about a stable minimum
of the potential V(x) is

(A) 20 (B) 20

(C) 40 (D) 4 20

11. A particle of mass m is in a cubic box of size a. The potential inside the box (0  x < a, 0  y < a, 0  z <
142 2
a) is zero and infinite outside. If the particle is in an eigenstate of energy E  , its wavefunction is
2ma 2
3/ 2 3/ 2
2 3x 5y 6z 2 7x 4y 3z
(A)     sin sin sin (B)     sin sin sin
a a a a a a a a
3/ 2 3/ 2
2 4x 8y 2z 2 x 2y 3z
(C)     sin sin sin (D)     sin sin sin
a a a a a a a a

12. Let v, p and E denote the speed, the magnitude of the momentum, and the energy of a free particle of rest mass
m. Then
dE
(A)  constant (B) p = mv
dP

cp
(C) v  (D) E = mc2
2 2 2
p m c

 
13. An electric field E  r   ˆr   sin  cos ˆ exists in space. What will be the total charge enclosed in a sphere of
 
unit radius centered at the origin ?
(A) 40 (B) 40 ( + )
(C) 40 ( – ) (D) 40


14. The magnetic field associated with the electric field vector E  E 0 sin(kz  t)ˆj is given by
 E  E0
(A) B   0 sin  kz  t  ˆi (B) B  sin  kz  t  ˆi
c c
 E0  E0
(C) B  sin  kz  t  ˆj (D) B  sin  kz  t  kˆ
c c

Contact Us : Website : www.eduncle.com | Email : support@eduncle.com | Call Us : 7231923276 4


Physics (MSP)

15. An electromagnetic wave is given by


1   z 
Ex  cos  4  107  t    ; E y  0;
2   c 
Ez = 0 then —
(A)  = 15 m is plane polarised and propagates in z-direction
(B)  = 15 m is plane polarised and propagates in x-direction
(C)  = 15 m is plane polarised and propagates in y-direction
(D)  = 51 m is plane polarised and propagates in z-direction

16. If the binding energy B of a nucleus (mass number A and charge Z) is given by
(2Z  A) 2 a C Z2
B  a V A  a S A 2/3  a sym  1/3
A A
where aV = 16 MeV, aS = 16 MeV, asym = 24 MeV and aC = 0.75 MeV, then for the most stable isobar for a nucleus
with A = 216 is
(A) 68 (B) 72
(C)  (D) 92

17. Equipotential surface corresponding to a particular


 charge distribution are given by 4x2 + (y – 2)2 + z2 = Vi where
the value of Vi are constants. The electric field E at the origin is
 
(A) E  0 (B) E  2xˆ
 
(C) E  4yˆ (D) E  4yˆ

18. The two dimensional lattice of graphene is an arrangement of Carbon atoms forming a honeycomb lattice of lattice
spacing a, as shown below. The Carbon atoms occupy the vertices.

c1
d2
c2
a1 d1 b1
a2 b2
a

The Wigner-Seitz cell has an area of


3 2
(A) 2a2 (B) a
2
2 3 3 2
(C) 6 3a (D) a
2

19. Experimental measurements of heat capacity per mole of Aluminum at low temperatures show that the data can
be fitted to the formula, CV = aT + bT3, where a = 0.00135 JK–2 mole–1, b = 2.48 × 10–5 JK–4 mole–1 and T is the
temperature in Kelvin. The entropy of a mole of Aluminium at such temperatures is given by the formula
b 3 aT b 3
(A) aT + T + c, where c > 0 is a constant (B)  T + c, where c > 0 is a constant
3 2 4
b 3 aT b 3
(C) aT + T (D)  T
3 2 4

Contact Us : Website : www.eduncle.com | Email : support@eduncle.com | Call Us : 7231923276 5


Physics (MSP)

a b
20. The potential of a diatomic molecule as a function of the distance r between the atoms is given by V(r)   6
 12 .
r r
The value of the potential at equilibrium separation between the atoms is:
(A) –4a2/b (B) –2a2/b
(C) –a2/2b (D) –a2/4b

21. A gas of molecules each having mass m is in thermal equilibrium at a temperature T. Let vx, vy, vz be the Cartesian
components of velocity, v , of a molecules. The mean value of (vx – vy + vz)2 is

2 2 k BT 2 2 k BT
(A) 1      (B) 1     
m m
2 2 k BT 2 k BT
(C)      (D)   2 
m m

1
22. The energy eigenvalues of a particle in the potential V(x) = m2x2 – ax are
2

 1 a2  1 a2
(A) En   n     (B) En   n    
 2 2m2  2 2m2

 1 a2  1
(C) En   n     (D) En   n   
 2 m2  2

23. A particle of rest mass m0 is moving uniformly in a straight line with relativistic velocity c, where c is the velocity
of light in vacuum and 0 <  < 1. The phase velocity of the de Broglie wave associated with the particle is
c
(A) c (B) 

c
(C) c (D) 2

24. A photon of wavelength  is incident on a free electron at rest and is scattered in the backward direction. The
functional shift in its wavelength in terms of the Compton wavelength c of the electron is
C 2 C
(A) (B)
2 3
3 C 2 C
(C) (D)
2 

25. Three identical non-interacting particles, each of spin ½ and mass m, are moving in a one-dimensional infinite
potential well given by,
 0 for 0  x  a
V(x)  
 for x  0 and x  a
The energy of the lowest energy state of the system is
 2 2 2  2 2
(A) (B)
ma 2 ma 2
32 2 5 2  2
(C) (D)
ma 2 2ma 2

Contact Us : Website : www.eduncle.com | Email : support@eduncle.com | Call Us : 7231923276 6


Physics (MSP)

26. The degree of polarization for ordinary light reflected from glass (index = 1.5) at an angle incident at 45° is —
(A) 83.3% (B) 6.7%
(C) 28.1% (D) 61.9%

1 1 1
 
1 1 1
27. Consider the matrix M = 
1 1 1
 
The eigenvalues of M are
(A) 0, 1, 2 (B) 0, 0, 3
(C) 1, 1, 1 (D) –1, 1, 3

28. The instantaneous position x(t) of a small block performing one-dimensional damped oscillations x(t) = Ae–rt cos(t + a).
Here  is the angular frequency,  the damping coefficient, A the initial amplitude and  the initial phase. If x|t=0 = 0 and
dx
= v, the values of A and  (with n = 0, 1, 2, ...) are
dt t  0
v (2n  1) v
(A) A  ,  (B) A  ,   n
2 2 
v (2n  1) 2v (2n  1)
(C) A  ,   (D) A  ,
 2  2

29. A quarter-wave plate is placed in between a polarizer and a photo-director. When the optic axis of the quarter-
wave plate is kept initially parallel to the pass axis of the polarizer and perpendicular to the direction of light propagation.
The intensity of light passing through the quarter-wave plate is measured to be I0 (see figure). If the quarter wave
plate is now rotated by 450 about an axis parallel to the light propagation, what would be the intensity of the emergent
light measured by the photo-director ?
Direction of
Rotation of
Quarter Wave
Plate

Polaroid Quarter Wave Plate Photo - Detector


I0 I0
(A) (B)
2 2
I0
(C) (D) I0
2 2

 2 k(2k  3)
30. In a crystalline solid, the energy band structure (E-k relation) for an electron of mass m is given by E  .
2m
The effective mass of the electron in the crystal is
2
(A) m (B) m
3
m
(C) (D) 2m
2
Contact Us : Website : www.eduncle.com | Email : support@eduncle.com | Call Us : 7231923276 7
Physics (MSP)

Section-(B) Multiple Select Questions (MSQ)

1. Which of the followings are not the values of RB and RC for the transistor inverter given below if ICsat = 10mA ?
Vi Vi

10V 10V

OV t t
Vcc = 10V

RC
Vc
RB
Vi
hfE = 250

(A) RB = 150 , RC = 1  (B) RB = 150 K, RC = 1 K


(C) RB = 150 M, RC = 1 M (D) RB = 0.15 , RC = 0.1 

2. A system of non-interactive Fermi particles with Fermi-energy EF has density of states proportional to E , where
E is the energy of a particle. The average energy per particle at temperature
T = 0 is not equal to:
1 1
(A) EF (B) EF
6 5
2 3
(C) EF (D) EF
5 5

3. Properties of semiconductor are :


(A) Behaves like an insulator at 0 kelvin
(B) Resistivity more than conductors but less than insulators
(C) Resistivity less than conductors but more than insulators
(D) Resistance decreases with the rise in temperature and vice-versa.

4. A periodic function can be expressed in a Fourier series of the form,



f (x)    a n cos  nx   b n sin  nx  
n 0

The functions f1(x) = cos2 x and f2(x) = sin2x are expanded in their respective Fourier series. If the coefficients for
the first series are an(1) and bn(1), and the coefficients for the second series are an(2) and bn(2), respectively, then which
of the following(s) is/are incorrect ?
(1) 1 1 1 1
(A) a 2  and b (2)
2 
(1)
(B) b 2  and a (2)
2 
2 2 2 2
(1) 1 1 1 1
(C) a 2  and a (2)
2 
(1)
(D) b 2  and b (2)
2 
2 2 2 2

Contact Us : Website : www.eduncle.com | Email : support@eduncle.com | Call Us : 7231923276 8


Physics (MSP)

5. Choose the correct statement :


(A) Bosons have integer spin (B) Bosons obey pauli exclusion principle
(C) Fermions obey pauli exclusion principle (D) Fermions have Integer spin

6. Which of the following statements is CORRECT ?


(A) Indistinguishable particles obey Maxwell-Boltzmann statistics
(B) All particles of an ideal Bose gas occupy a single energy state at T = 0
(C) The integral spin particles obey Bose-Einstein statistics
(D) Protons obey Fermi-Dirac statistics

7. Consider free expansion of one mole of an ideal gas in an adiabatic container from volume V1 to V2. The entropy
change of the gas, calculated by considering a reversible process between the original state (V1, T) to the final state
(V2, T) where T is the temperature of the system is denoted by S1. The corresponding change in the entropy of
the surrounding is S2. The corresponding change in the entropy of the surrounding is S2. Which of the following
combinations are incorrect?
(A) S1 = R In(V1 / V2), S2 = –R In (V1 / V2) (B) S1 = – R In(V1 / V2), S2 = R In (V1 / V2)
(C) S1 = R In(V2 / V1), S2 = 0 (D) S1 = –R In(V2 / V1), S2 = 0

8. For a liquid to vapour phase transition at Ttr, which of the following plots between specific Gibbs free energy g
and temperature T are incorrect?
g g

Liquid Vapour Liquid Vapour


(A) (B)

T T
Ttr Ttr
g g

Liquid Vapour Liquid Vapour


(C) (D)

T T
Ttr Ttr

9. In presence of a magnetic field Bjˆ and an electric field (–E) k̂ , a particle moves undeflected. Which of the following
statements is (are) correct?
E E
(A) The particle has positive charge, velocity = – î (B) The particle has positive charge, velocity = î
B B
E E
(C) The particle has negative charge, velocity = – î (D) The particle has negative charge, velocity = î
B B

Contact Us : Website : www.eduncle.com | Email : support@eduncle.com | Call Us : 7231923276 9


Physics (MSP)

10. Which of the following circuits satisfy the Boolean expression AB  AB = F

A A
B B
F F
(A) (B)

A A
B B
F F
(C) (D)

Section-(C) Numerical Answer Type Questions (NAT)

1. Consider the op-amp circuit shown in the figure.


If the input is a sinusoidal wave Vt = 5sin(1000t), then the amplitude of the output V0 is.......sin t
1 F

1K
1K
Vi – V0
+

2. If the average life time of an excited state of hydrogen is of the order of 10–8 s, estimate how many rotation an
electron makes when it is in the state n = 2 and before it suffers a transition to state n = 1. Bohr radius = 5.3 ×
10–11 m.(In terms of 106)

3. The static dielectric constant of water is 8.1, and its refractive index is 1.33 calculate the percentage contribution
of ionic polarizability ?

4. If S denotes the surface of the sphere r = (a sin cos, a sinsin, a cos), where 0 , 0 2, Evaluate
2
 (x  y 2 )dS .
S

5. Two electric dipoles P1 and P2 are placed at (0, 0, 0) and (1, 0, 0) respectively with both of them pointing in the
+z direction. Without changing the orientations of the dipoles P2 is moved to (0, 2, 0). The ratio of the electrostatic
potential energy of the dipoles after moving to that before moving is ________.

6. A tiny dust particle of mass 1.4 × 10–11 kg is floating in air at 300K. Ignoring gravity, its rms speed (in µm/s) due
to random collisions with air molecules will be closest to ____(in terms of 10-6).

Contact Us : Website : www.eduncle.com | Email : support@eduncle.com | Call Us : 7231923276 10


Physics (MSP)

7. A particle is confined in a one dimensional box with impenetrable walls at x = ±a. Its energy eigenvalue is 2 eV
and the corresponding eigenfunction is as shown below.

–a 0 +a
The lowest possible energy of the particle is _______(in eV).


8. For a wave in a medium the angular frequency  and the wave vector k are related by, 2 = (02 + c2 k2) where
0 and c are constants. The product of group and phase velocities, i.e vg.vp is nc2 then the value of n will be ________.

9. A relativity correction to GPS. GPS satellites move at about 4km/s = 4000 m/s. Show that a good GPS receiver
need to correct for time dilation if it is to produce results consistent with atomic clocks accurate to 1 part in 1013.(in
terms of 10–10)

10. An object is placed at an of 45 cm from a converging lens of focal length 30 cm. A mirror of radius of curvature
40 cm is to be placed on other side of lens so that the object coincides with its image. The distance of mirror from
lens if mirror is convex_____ 10–10 cm.

11. A 250-g mass is mounted on a spring with a spring constant of k = 3.3 N/m. The damping constant for this system
is b = 8.4 × 10–3 kg/s. How many oscillations will the system undergo during the time it takes the amplitude to decay
to 1/e of its original value?

12. In an intrinsic semiconductor, the free carrier concentration n (in cm–3) varies with temperature T (in Kelvin) as
shown in the figure below. The band gap of the semiconductor is _________ (in eV) (use Boltzmann constant kB =
8.625 × 10–5 eVK–1).

36.5
35.5
In(n) 34.5

33.5
32.5

2 3 1000/T

13. The first order diffraction peak of a crystalline solid occurs at a scattering angle of 30° when the diffraction pattern
is recorded using an x-ray beam of wavelength 0.15 nm. If the error in measurements of the wavelength and the angle
are 0.01 nm and 1° respectively, then the error in calculating the inter-planar spacing will approximately be________nm

14. The value of i  i , where i = 1, is ______.

15. Let S be the boundary of the region consisting of the parabolic cylinder z = 1 – x2 and the planes y = 0, y = 2
  2
and z = 0. Then value of the integral  F.nˆ dS.
 where F  xy ˆi  (y 2  e xz )ˆj  sin(xy) kˆ and nˆ is the outwardly
S
drawn unit normal to S.

Contact Us : Website : www.eduncle.com | Email : support@eduncle.com | Call Us : 7231923276 11


Physics (MSP)

16. Electric field component of an electromagnetic radiation varies with time as E = a(cos 0t + sin t cos 0t), where
a is a constant and the values of  and 0 are 1 × 1015 s–1 and 5 × 1015 s–1 respectively. This radiation falls on a metal
of work function 2eV. The maximum kinetic energy (in eV) of photoelectrons is __________.

17. A variable power supply 5V – 20V is connected to a Zener diode specified by a breakdown voltage of 10V (see
figure). The ratio of the maximum power to the minimum power dissipated across the load resistor is ________.
500

5V – 20V 1 k

18. In an ideal operational amplifier depicted below, the potential at node A is


25 k
5 k A

5v
+
IV

19. 1011 binary input have been applied at X3X2X1X0 input in the shown logic circuit made of XOR gates. The binary
output Y3Y2Y1Y0 of the circuit will be
X3 Y3

X2 Y2

X1 Y1

X0 Y0

20. Find the constant  so that F is conservative vector field where F = (xy – z3)i + ( – 2)x2 j + (1 – )xz2 k.

Contact Us : Website : www.eduncle.com | Email : support@eduncle.com | Call Us : 7231923276 12


Physics (MSP)

Answer Key

Section-(A) Multiple Choice Questions (MCQ)


1 2 3 4 5 6 7 8 9 10
A A B C C B D C C B
11 12 13 14 15 16 17 18 19 20
D C A A A C D D A D
21 22 23 24 25 26 27 28 29 30
A A B D C A B C B C

Section-(B) Multiple Select Questions (MSQ)


1 2 3 4 5 6 7 8 9 10
A,C,D A,B,C A,B,D A,B,D A,C B,C,D A,B,D B,C,D B,D A,B,C

Section-(C) Numerical Answer Type Questions (NAT)


1 2 3 4 5 6 7 8 9 10
3.53 8.22 70 0 0.125 30 0.5 1 1 50
11 12 13 14 15 16 17 18 19 20
34 0.69 0.011 0 8 1.28 9.2 0 1101 4

Contact Us : Website : www.eduncle.com | Email : support@eduncle.com | Call Us : 7231923276 13


Physics (MSP)

Sample Questions
Section-(A) Multiple Choice Questions (MCQ)

1. (A)
Vo

2r
r
A
f
Velocity at the centre = V0 ...(1)
V0
and the angular velocity about the centre = ...(2)
2r
Thus, V0 > .wor, The sphere slips forward and thus the friction by the plane on the sphere will act backward. As
the friction is kinetic, its value is
N = mg, ...(3)
and the sphere will get decelerated by acm = f/m
ft
Hence, V(t) = V0 – ...(4)
m
This friction will also have a torque  = fr about the centre. ...(5)
and  = I ...(6)
From equation (5) and (6) we get
I = f.r
f .r f .r
 
I 2 / 5mr 2
5f
 ...(7)
2mr
and (t) = o + t
V0 5ft 2 V 2 ft
   r  0   ...(8)
2r 2mr 3 2 5 m
adding (4) and (8)
2 V
V r  0  V0
5 5
Vo 5ft
V(t) =  ...(9)
2 2m
also V = r (pure rolling)
6
V V0
7
Pure rolling starts when V(t) = r(t)

2. (A) The gaussian condition requires that the image charge must be situated inside the sphere.
Boundary condition that potential at any point on the surface is zero.
P
r1
R
P1
P2 q
a b q’
d

Contact Us : Website : www.eduncle.com | Email : support@eduncle.com | Call Us : 7231923276 14


Physics (MSP)

At point P1
q q
 0 ...(1)
4 0 (d  R) 4 0 (R  b)
At point P2
q q
 0 ...(2)
4 0 (d  R) 4 0 (R  b)
From equation (1)
q q 

4 0 4 0 (R  b)

q(R  b)
q  ...(3)
(d  R)
Substituting equation (3) in equation (2) we get
q q(R  b) / (d  R)
 0
4 0 (d  R) 4 0 (R  b)

q q(R  b)

(d  R) (d  R)(R  b)
–2R2 + 2bd = 0
R2
or b =
d
 R2 
q  R  
and  q '  q(R  b)   d 
 (d  R)  (d  R)
qR
q'  ...(a)
d

3. (B) The transformer works on the principle of mutual induction.


Hence the emf induced in the secondary coil is linked with change in current in the primary coil.
di p
S   M
dt
di p
S  M
dt
di p
s 
dt

4. (C) Initial momentum


E2 – p2 c2 = m2 c4
p2 c2 = (2mc2)2 – m2 c4
p2 c2 = 3m2 c4
p = 3 mc
Initial energy
2mc2 + mc2 = 3mc2
Each is conserved so energy is 3 mc2, final momentum is 3 mc

Contact Us : Website : www.eduncle.com | Email : support@eduncle.com | Call Us : 7231923276 15


Physics (MSP)

2
E2 – p2 c2 = (3mc2)2 –  3 mc .c2

= 6 m2 c4 = M2 c2
M = 6 m = 2.5 m
In this process some kinetic energy was converted into rest energy, so
M > 2m

pc 2 3 mc.c 2
v 
E E

c
v
3

5. (C) Ixy = –  m i x i yi = 0, Ixz = –  m i x i zi = 0, Iyz = –  mi yi zi = 0


i i i

1000  3600  v
6. (B) = 1.08 × 1012 × 1000
v2
1 2
c

105  36  v v c
 1080  1012  c v
2 2
v v 2
1 1
c2 c2

d2 x GMm d2 x GM A
7. (D) m     2  2
dt 2 x2 dt 2 x x
dv  A dx d  v2  d  A  v2 A
v  2       C
dt x dt dt  2  dt  x  2 x
A
when x = R, v = 0, then c = –
R
v2 A A 1 1 dx 2A R  x
   v  2A   
2 x R x R dt R x
0 t
x 2A
 dx   dt
R Rx 0
R
2
v A A 1 1 dx 2A R  x
   v  2A   
2 x R x R dt R x
0 t
x 2A
 dx   dt
R Rx 0
R

Put x  u 2  dx  2udu and x  0, u  0 and also, x  R, u  R


0 t 0
2u 2 2A u R u  2A
 du   dt  2  R  u 2  sin 1   t
R R u 2
0 R 2 2 R R R

 R R 1 R  2A R 2A
 2  RR  sin  t  2  sin 1 1  t
 2 2 R R 2 R

2A R  R R
t  2   t  
R 2 2 2 2A

Contact Us : Website : www.eduncle.com | Email : support@eduncle.com | Call Us : 7231923276 16


Physics (MSP)

1 R 3 2
t ...(1)
2 2 GM
mv 2 GMm 2 GM 2R 42 R 2 GM 42 R 3
and   v  v      2
R R2 R  2 R GM
R 3 2 R 3 2 
2   ...(2)
GM GM 2
1  
From (1) and (2), t = 
2 2 2 4 2

8. (C) We known
E 0r n 2  n1

E 0i n 2  n1
E 0t 2n1

E 0i n 2  n1
 n2 
1  n 
1
E
 or  E 0i
 
n
 2  1
n 
 1 
1.5  1 0.5
 2  2 
1.5  1 2.5
2
 V/m
5
2
and for E 0t  E 0i 
n
1 2
n1
1
 2 2  2
5
8
E 0t  V / m
5

9. (C) Group velocity


d
vg 
dk
1 dk d  
or v  d  d  v 
g  
since  = vk
1 1  dv
 
vg v v 2 d
c
Also v 
n
1 n n 2 d  c 
  2  
vg c c d  n 
n  dn
 
c c d
c
or vg 
dn
n
d
Contact Us : Website : www.eduncle.com | Email : support@eduncle.com | Call Us : 7231923276 17
Physics (MSP)

1 a
10. (B) V(x) = m02 x 2 
2 2mx 2
dV a a
 m02 x   0  x4  2 2
dx mx 3 m 0

d2V 3a 3m2 02


 m02   m2
0   4m02
dx 2 x  x0
mx 4 m

d2 V
dx 2 x  x0 4m02
   20
m m

2 2 2 2 2 142 2
11. (D) E n x ,n y ,n z   n x  n y  n z  
2ma 2 2ma 2
 n 2x  n 2y  n 2z  14  n x  1, n y  2, n z  3.

mv m2 v2 p2 v2
12. (C) p  m ' v   p2   m 2 2
v  p 2
 , m  rest mass energy
v2 v2 c2
1 1 2
c2 c

 p2  p2 pc
 v2  m2  2   p2  v2  2 2 2
v
 c  m c p p  m2 c2
2

2
c
 
13. (A) Qenc = 0 
 E·da  0  ˆr   sin  cos ˆ · r 2 sin d d rˆ  4 0
  
 
 k  E kzˆ  E 0 sin(kz  t)yˆ kE E
14. (A) B     0 sin  kz  t  xˆ   0 sin  kz  t  xˆ
   c

15. (A) In z direction wave


Ex = E0 cos (t – kz)
Propagating in z direction
1
E0  V/m
2
 = 4 × 107 rad/sec.
4  107
k 1/ m
c
We know
2
k

2 2  3  108
or   
k 4  107
= 15 m
Electric field component

E  E x ˆi, in x  direction
Wave x-direction (X-Z plane) is plane polarised and propagates in z-direction.

Contact Us : Website : www.eduncle.com | Email : support@eduncle.com | Call Us : 7231923276 18


Physics (MSP)

dB 2(2Z  A)  2 2a C Z
16. (C) For the most stable isobar for a nucleus  0  a sym  1/3  0
dZ A A
2(2Z  216)  2 2Z 4(2Z  216) 3 2Z
 24 216
 0.75
(216)1/3
0
9

4 6
0

4(2Z  216) Z
   0  16(2Z  216)  9Z  0  41Z  216  16  Z  82.3
9 4
  
17. (D) E  V  8xxˆ  2  y  2  yˆ  2zzˆ  E  0,0   4yˆ

 
18. (D) Primitive lattice vectors are b1 and b 2
 3
b1  3a cos 30 ˆi  3a cos 60 ˆj  a( 3iˆ  ˆj)
2
 3   3 3 2
b2  a( 3iˆ  ˆj),  A  b 2  b1  a
2 2

c v dT aT  bT 4 b
19. (A) ds    s dT  aT  T 3  c
T T 3

a b V a 12b 1 12b 
20. (D) V(r)    , for equilibrium  0  (6) 7  13  0  7  6a  6   0
r 6 r12 r r r r  r 
1 1
12b  12b  6  2b  6
 6a  6  0  r    r 
r  6a   a 
1
 
  2b  6  a b a2 a2 a2
 V r           .
  a    2b   2b  2 2b 4b 4b
     
 a   a 

21. (A) (vx – vy + vz)2 = vx2 + 2vy2 + 2vy2 – 2vxvy + 2vzvx – 2vyvz

(v x  v y  v z ) 2  v 2x   2 v2y  2 v 2y  2 v x v y  2 v z v x  2 v y vz

k BT
v 2x  v 2y  v 2z  and v x  v y  v z  0
m
k BT
(v x  v y   v z ) 2  1   2  2 
m

p 2x 1
22. (A) Hamiltonian (H) of Harmonic oscillator, H   m2 x 2
2m 2

 1
Eigenvalue of this, E n   n   
 2

p 2x 1 p2 1  2ax a2  a2
But here, H   m2 x 2  ax  H  x  m2  x 2  2
 2 4
2m 2 2m 2  m m   2m2
2
p2 1  a  a2
H  x  m2  x  2 

2m 2  m  2m2

 1 a2
Energy eigenvalue, E n   n    
 2 2m2

Contact Us : Website : www.eduncle.com | Email : support@eduncle.com | Call Us : 7231923276 19


Physics (MSP)

23. (B) E2 = p2c2 + m02c4


dE E c2 c2 c
2E  2pc 2  E.vg  pc 2    vp  vp  
dp p vg c 

24. (D)  = c(1 – cos )


When photon scatter in backward direction then
 =  so  = 2c
Functional shift is
 2 C

 

1
25. (C) Spin s = means particles are fermions and it will obey Pauli Exclusion Principle. Degeneracy g = 2s + 1
2
 g = 2 means in every state maximum 2 identical particle can be adjusted. If we have three fermions, then in ground
state two fermions will adjust and in next higher level one fermion will adjusted. Thus the energy of the lowest energy
state of the system is
2 2 42 2 62 2 32 2
2 ×   
2ma 2 2ma 2 2ma 2 ma 2

26. (A) According to Snell's law


sin i
 1n 2
sin T
But here i = 45° and 1n2 = 1.5
sin i
sin T 
1n2

sin 45 1
 
1.5 2 1.5

1  1 
 T  sin    28.1
 2.121 

sin 2 (i  T ) sin 2 (16.9)


 R   
sin 2 (i  T ) sin 2 (73.1)
(.2924) 2
  0.93392
(.9568) 2

tan 2 (i  T ) tan 2 (16.9)


and R II  
tan 2 (i  T ) tan 2 (73.1)
(3.038) 2
  0.008526
(3.291) 2

R   R II
So, P( ) 
R   R II
0.084866
  0.83268
0.101918
i.e., The deg ree of polarization is 83.3%

Contact Us : Website : www.eduncle.com | Email : support@eduncle.com | Call Us : 7231923276 20


Physics (MSP)

1   1 1 
 1 1  1   0
27. (B) For eigen values 
 1 1 1   
(1 – ) ((1 – )2 – 1) – (1 –  – 1) + 1(1 – (1 – )) = 0
(1 – ) (1 + 2 – 2 – 1) +  +  = 0  2 – 2 – 3 + 22 + 2 = 0
3 – 32 = 0  2( – 3) = 0   = 0, 0, 3

(2n  1)
28. (C) x|t=0 = A cos  = 0  cos = 0   = 
2
dx
 Ae  t    cos  t     Ae t sin  t    
dt
= Ae–t (– cos(t + ) + sin(t + ))
dx v
 v  A   cos      sin      v  A  v  A 
dt t 0 

29. (B) After passing through half wave plate the plane polarized light of intensity I0 will remain in early polarized
rotated at 45° but the intensity is driven by Malus law
I0
I  I0 cos 2 ()  I0 cos 2 (45) 
2

30. (C) The expression of effective mass of electron in solid is

2
m =
d2 E 2
dk
dE  2 d2 E  2 2 2 m
  4k  3  2   4    m 
dk 2m dk 2m m 2

Section-(B) Multiple Select Questions (MSQ)

1. (A,C,D)
At saturation
VCC
Icsat = R
C

VCC 10V
RC = I  10mA = 1k
csat

At saturation:
Icsat 10mA
IB =   250
d.c

IB = 40 A
Choosing IB = 60 A to ensure saturation and using
Vi  0.7V
IB = RB
Vi  0.7V 10V  0.7V
RB = 
60A 60A
RB = 155 k
Contact Us : Website : www.eduncle.com | Email : support@eduncle.com | Call Us : 7231923276 21
Physics (MSP)

Choosing RB = 150 k which is standard value, then


Vi  0.7V
IB = RB = 62 A

I csat
and IB = 62 A > 
d.c

= 40 A
 Use R = 150 k and RC = 1 k

2. (A,B,C)
In the three dimensional case, the density of the fermi-particles is proportional to E.
EF
2
 E  k  4k dk
0
 E  EF
2
 (k) 4k
0
dk.

k 3
2k2  L  2
0 2m  2   4k dk
 k 3
 L  2
0  2  ·4k dk
k k
4 k5
2 k
0
dk.
2 5 0
 k
 . 3
2m 2 2m k 3
 k dk
0 3 0

2 2 2
3 3 k 
 k2  

 
2m 5 5  2m 
3
 EF
5

3. (A,B,D)

4. (A,B,D)
1 1 1 1
f1(x) =  cos 2x and f2(x) =  cos 2x
2 2 2 2
(1) 1 (2) 1
Hence, a 2  and a 2  
2 2
All the bn’s of each of the series are zero. As there is no sine terms in any of the two given functions.

5. (A,C)

6. (B,C,D)
Distinguishable particles obey Maxwell-Boltzmann statistics.

7. (A,B,D)
Free expansion is irreversible process when gas expand V1 to V2 which can be explained by choosing any path between
two state (because entropy is state function). So one can choose reversible isothermal process.
V2
So S2 = R In V .
1

Hence it is free expansion so entropy of surrounding is S2 = 0

Contact Us : Website : www.eduncle.com | Email : support@eduncle.com | Call Us : 7231923276 22


Physics (MSP)

8. (B,C,D)

9. (B,D)
     E
 F  q  E  v  B   0  v 
  B
  E
For +ve charge : a   kˆ  v  xˆ
B
  E
For –ve charge : a  kˆ  v  xˆ
B

10. (A,B,C)
(A) F  AB  AB
(B) F   A  B  .AB   A  B  . A  B  AB  AB
 
(C) F   A  B  . A  B  AB  AB
 
(D) F  AB  AB

Section-(C) Numerical Answer Type Questions (NAT)

1. 3.53
v0 X R F XC 103 1
  F , XF   where R F  1  103 , XC 
vin R1 R F  X C (1  j) j  10  106
3

v 0 103 1 1 5 5 2
  3   v0  sin t  sin t
v in 2 10 2 2 2

2. 8.22
Average lifetime of an excited state of hydrogen = 10–8 sec
Bohr radius, r = 5.3 × 10–11 m.
Velocity of electron in the nth orbit of hydrogen atom,

v1 2.19  106
vn   m/s
n n
2.19  106
If, n = 2, vn = m/s
n
Radius of n = 2 orbit,

rn  n 2 r1  4  Bohr radius
 rn = 4 × 5.3 × 10–11m
Number of revolutions made in 1 sec
vn 2.19  106
 
2r 2  2  4  5.3  1011
Therefore, number of revolutions made in 10–8 sec,
2.19  106  108

2  2  4  5.3  1011
= 8.22 × 106 revolutions.

Contact Us : Website : www.eduncle.com | Email : support@eduncle.com | Call Us : 7231923276 23


Physics (MSP)

3. 70
According to Clausius-Mosotti relation we have
r 1 N( e   i )
 ...(1)
r 2 3 0
Where e and i are electronic and ionic polarizabilities and (e + i) is the total probability.
The refraction index n is given by the relation
n 2  1 N e
 ...(2)
n 2  2 3 0
Dividing equation (2) by equation (1), we get
n 2  1 r 2 e
2
 
n  2 r 1  e   i
e i
But, 1        
e i e i

Thus percentage contribution of ionic polarizability is


i
  100
 e  i

  n 2  1  r 2    100
 1  2
  n  2  r 1 
 
2
 1.33  1 8.1  2  
 

 1   100
2

1.33  2  8.1  1 
 
 
= 70%

4. 0
The unit normal to the sphere will be directed away from the centre, which is origin. Thus
xi  yj  zk i a sin  sin   k a cos 
n̂  
2 2 2
(x  y  z a sin  cos 2   a 2 sin 2   a 2 cos 2 
2 2

= i sin cos + j sin sin + k cos


 
Also dS  r0  r dd
= |(acoscos, acossin, –a sin ) × (–a sin sin, asincos, 0)| dd
= a2sin d d.
and x2 + y2 = a2sin2.
2 2 2 2
 I  S (x  y )dS  S (x  y )ndS
2x x
  a 4 sin 3 (iˆ sin  cos   ˆjsin  sin   kˆ cos )dd.
0 0

x 2x x 2x 2x x
 ˆi   a 4 sin 4  cos dd  j  a 4 sin 4  sin dd  kˆ   a 4 sin 3  cos dd.
0 0 0 0 0 0

2x 2x
But  cos d   sin d  0, giving first and second term of the integral equal to zero and the third term is
0 0

2x x
  a 4 sin 3  cos dd  0
0 0

 The given integral = i.0 + j.0 + k.0 = 0.

Contact Us : Website : www.eduncle.com | Email : support@eduncle.com | Call Us : 7231923276 24


Physics (MSP)

5. 0.125
Electrostatic potential energy
1 U 2 r13 1
U     = 0.125
r3 U1 r23 8

6. 30

3kT 3  1.38  1023  300


Vrms    30  106 m / s
m 1.4  1011

7. 0.5
The given state is representation of first exited state whose energy is 2eV.
If En is energy of nth state and E0 is energy of ground state then
En = n2 E0.
So, E2 = 4E0 = 2eV
 E0 = 0.5eV

8. 1
2 = (02 + c2 k2)
d  d
2.  2c 2 k  .  c 2  v p .v g  c 2
dk k dk

9. 1
Let us calculate the magnitude of the time dilation effect by inserting v = 4000 m/s into Eq. 26–1 a:
1 1
t  t 0  t 0
2
v  4  103 m / s 
1 2 1  8 
c  3  10 m / s 
1
 t 0 .
1  1.8  1010
1
We use the binomial expansion: (1 ± x)n  1 ± nx for x << 1(see Appendix A–5) which here is (1 – x)–1/2  1 + x.
2
That is
1
t = (1 + (1.8 × 10–10)) t0 = (1 + 9 × 10–11) t0.
2
The time "error" divided by the time interval is
( t  t 0 )
 1  9  1011  1  9  1011  1  10 10
t 0
Time dilation, if not accounted for, would introduce an error of about 1 part in 1010, which is 1000 times greater than
the precision of the atomic clocks. Not correcting for time dilation means a receiver could give much poorer position
accuracy.

10. 50
The object and image will coincide only if light ray retraces its path and it will occur only when the ray normally strikes
at mirror.
In other words, the centre of curvature of mirror and rays incident on mirror are collinear.

Contact Us : Website : www.eduncle.com | Email : support@eduncle.com | Call Us : 7231923276 25


Physics (MSP)

O
O, 1

45 cm 40 cm
The rays after refraction from lens must be directed towards the centre of curvature of mirror at C.
If x is separation, then for lens.
u = –45 cm, v = x + 40, f = 30 cm
Using lens formula,
1 1 1
 
v u f
1 1 1
or  
x  40 45 30
45(30)
or x = – 40 = 50 cm
45  30

11. 34
First, need to check whether the system is underdamped, critically damped, or overdamped, by comparing b with
2m0.
k
2m0  2m  2 km  2 0.25 kg  3.3 N / m  1.8kg / s
m
With b = 8.4 × 10–3 kg/s, b << 2m0, and so the motion is underdamped and will oscillate with :
 b 
x(t)  A exp   t  cos(t  )
 2m 

b2
  20 
4m 2
Asked for the number of oscillations before the amplitude decays to 1/e of its original value i.e.
 b 
A exp   t   A exp(1)
 2m 
Solve for the time t at which this happens:
b
t 1
2m
2m 2(0.25kg)
t   59.5s
b 8.4  103 kg / s
Also need to calculate the period for one oscillation:
2 2 2
T  
2
 b k b2
02  
4m 2 m 4m 2
2
  1.73seconds
3.3 N / m (8.4  103 kg / s) 2

0.25kg 4(0.25kg) 2
Thus, the number of oscillations is 59.5/1.73 = 34.

Contact Us : Website : www.eduncle.com | Email : support@eduncle.com | Call Us : 7231923276 26


Physics (MSP)

12. 0.69
 Eg  n1  Eg  1 1 
Since ni = n = N c N v exp     exp     
 2kT  n 2  2k  T2 T1  
 n1   1 1  5
 Eg = 2k In  n   T  T   2  8.625  10  36.5  32.5  0.003  0.0002 
 2  2 1 

 Eg = 2 × 8.625 × 10–5 × 4/1 × 10–3 = 0.69eV

13. 0.011
Bragg's Law for n = 1,  = 2d sin 
 d 1 d  cos 
 d =   , 
2sin   2sin   2sin 2 
Error in d can be calculated as
2 2
 d   d   1  2   cos   2
d2    2    2       2  
       2sin    2sin  
2 2 2
2d 1  2sin   2   cos    2sin   2
 2
 2
         
d (4sin )     2sin  sin     
1

 d2 2  2   2     2  2
 2  2   d  d      
d  tan 2      tan   
where  = 30°,
 = 1.5 × 10–10 m,
 = 0.1 × 10–10 m,
 = 1°
 1.5  1010
d   1.5  1010 m
2sin  2sin 30
1
2
    2 1
2 2
 10   
10  0.1  10   180  3
  
Thus, d  1.5  10     1.5  1010 (0.067) 2   
 1.5  10
10   180  
  tan 30   
   
   
 
1 1
 1.5  1010 (0.067) 2  (0.03) 2  2  1.5  1010 [0.005389]2
d = 1.5 × 10–10 × 0.0734 = 0.11 × 10–10 = 1.1×10–11 m = 1.1 × 10–2 nm

14. 0
i  i  i  i   i i 2  i 2  i 2
i  i 
i  i i  i
ii ii
  0
i  i i  i

15. 8

 F.nˆ ds  ?

s

by divergence theorem

 F.nˆ ds   (.F) dv


s v
...(1)

Contact Us : Website : www.eduncle.com | Email : support@eduncle.com | Call Us : 7231923276 27


Physics (MSP)

x
2
where F  xy ˆi  (y 2  e xz ) ˆj  sin(xy) kˆ
.F = y + 2y = 3y
from equation (1)
1 2 1 x 2
 (.F)dv  
v
 
1 y  0 0
3y dx dy dz

1 2
  3y(1  x 2 )dx dy
1 y  0

2
1  3y2 
2
  (1  x )   dx
1
 2 0
1 3 
  (1  x 2 )   4  dx
1
 2 
1 1
 6 (1  x 2 )dx  12 (1  x 2 )dx
1 0

1
 x3  2
 12  x    12   8
 3 0 3

16. 1.28
Kmax =  – W
For given wave maximum kinetic energy is for highest 
so 0 = 5 × 1015 sec–1
6.6  1034 Js  5  1015 s 1 33  1019 J
   eV  3.28
2 6.28  1.6  1019
Kmax =  – W  3.28eV – 2eV = 1.28eV

17. 9.2
When V = 5V  open circuit voltage
1000
Vi = × 5 = 3.33 < VZ = 10V
1500
Vi2
 VL = Vi = 3.33V  PL,min = .
RL
When V = 20V  open circuit voltage
1000
Vi = × 20 = 13.33 > VZ = 10V
1500
2
VZ2 PL,max VZ2  10 
 VL  Vz  10  PL,max    2    9.2
R L PL,min Vi  3.33 

18. 0
Node A is virtually grounded.

Contact Us : Website : www.eduncle.com | Email : support@eduncle.com | Call Us : 7231923276 28


Physics (MSP)

19. 1101
1 X3 Y3 1

1 X2 Y2 1

0 X1 Y1 0

1 X0 Y0 1

20. 4
The necessary and sufficient condition that a force will be conservative is Curl F = 0

i j k
  
0
 x y z
xy  z 3 (  2)x 2 (1   )xz 2
 i(0 – 0) – j((1 – )z2 + 3z2) + k(2( – 2)x – x)) = 0
 –(– + 4)z2 j + ( – 4)x k = 0
  – 4 = 0   = 4

Contact Us : Website : www.eduncle.com | Email : support@eduncle.com | Call Us : 7231923276 29

You might also like